Anda di halaman 1dari 70

Dagdag v. Nepomuceno Juanson and had become private property.

In
G.R. No. L-12691. February 27, 1959 Sec 122 of the Land Registration Law, the
documents mentioned wherein lands are
Facts: A small parcel of land (Lot No. 3786), alienated, granted, or conveyed are
an alienable or disposable public land. documents transferring ownership, not
Covered by SalesPatent No. 257 was issued to documents of lease transferring ownership.
Margarita Juanson. The same land was also The Torrens Title of Dagdag must prevail.
issued by Lease No.49 executed by the Bureau
of Lands in favor of Andres de Vera. Juansons Chavez vs. Public Estate Authority and
Sales Patent was inscribed by the Register of AMARI
Deeds on July 11, 1927, and the Original Facts: The Public Estates Authority (PEA) is
Certificate of Title was issued to her. In the central implementing agency tasked to
1950,Simeon Dagdag bought it from the undertake reclamation projects nationwide. It
owner and the corresponding certificate of title took over the leasing and selling functions of
was given out. On the other hand, the lease to the DENR (Department of Environmental and
de Vera was transferred to Nepomuceno. Natural Resources) insofar as reclaimed or
Dagdags title and those of his predecessors about to be reclaimed foreshore lands are
contained no annotation of such lease concerned.
neither he had any knowledge of it. PEA sought the transfer to the Amari Coastal
Nepomuceno refused to surrender the land Bay and Development Corporation, a private
even in the face of Dagdags patent and title. corporation, of the ownership of 77.34
hectares of the Freedom Islands. PEA also
Issue: Who is entitled to the land and the sought to have 290.156 hectares of
products thereof? submerged areas of Manila Bay to Amari.
ISSUE: Whether or not the transfer is valid.
Ruling: The patents when registered in HELD: No. To allow vast areas of reclaimed
the corresponding Register of Deeds are lands of the public domain to be transferred to
indispensible. Weregard these as veritable Amari as private lands will sanction a gross
Torrens Title subject to no encumbrance violation of the constitutional ban on private
except those stated therein, plusthose corporations from acquiring any kind of
specified by the statutes, and lease is not one alienable land of the public domain.
of them. In addition, when the lease The Supreme Court affirmed that the 157.84
was renewedin 1949, the portion in question hectares of reclaimed lands comprising the
was no longer public land subject to the Freedom Islands, now covered by certificates
disposition of the Director ofLands because it of title in the name of PEA, are alienable lands
had already been granted to Margarita of the public domain. The 592.15 hectares of

Midterm Cases Exclusive Jurisdiction 1


submerged areas of Manila Bay remain Consolidated, Inc. as to Lots 1-5,Atok Big
inalienable natural resources of the public Wedge Corporation, as to Portions of Lots 1-5
domain. The transfer (as embodied in a joint and all of Lots 6-9, and by theRepublic of the
venture agreement) to AMARI, a private Philippines, through the Bureau of Forestry
corporation, ownership of 77.34 hectares of Development, as to lots 1-9.In support of the
the Freedom Islands, is void for being contrary application, both Balbalio and Alberto testified
to Section 3, Article XII of the 1987 that they had acquiredthe subject land by
Constitution which prohibits private virtue of prescription; Balbalio claimed to have
corporations from acquiring any kind of received Lots 1-5from her father shortly after
alienable land of the public domain. the Liberation.Benguet opposed on the ground
Furthermore, since the Amended JVA also that the June Bug mineral claim covering
seeks to transfer to Amari ownership of Lots 1-5 wassold to it on September 22, 1934,
290.156 hectares of still submerged areas of by the successors-in-interest of James Kelly,
Manila Bay, such transfer is void for being wholocated the claim in September 1909 and
contrary to Section 2, Article XII of the 1987 recorded it on October 14, 1909. From
Constitution which prohibits the alienation of the dateof its purchase, Benguet had been in
natural resources other than agricultural actual, continuous and exclusive possession
lands of the public domain. of theland in concept of owner, as evidenced
by its construction of adits, its affidavits of
annualassessment, its geological mappings,
REPUBLIC VS. CA AND DE LA ROSA geological samplings and trench side cuts, and
its payment of taxes on the land.For its part,
Facts: These consolidated cases arose from Atok alleged that a portion of Lots 1-5 and all
the application for registration of a parcelof of Lots 6-9 were covered bythe Emma and
land filed on February 11, 1965, by Jose de la Fredia mineral claims located by Harrison and
Rosa on his own behalf and on behalf of his Reynolds on December 25,1930, and recorded
three children, Victoria, Benjamin and on January 2, 1931, in the office of the mining
Eduardo. The land, situated in Tuding, recorder of Baguio.These claims were
Itogon,Benguet Province, was divided into purchased from these locators on November 2,
9 lots and covered by plan Psu-225009. 1931, by Atok, whichhas since then been in
According tothe application, Lots 1-5 were open, continuous and exclusive possession of
sold to Jose de la Rosa and Lots 6-9 to his the said lots asevidenced by its annual
children byMamaya Balbalio and Jaime assessment work on the claims, such as the
Alberto, respectively, in 1964.The application boring of tunnels, andits payment of annual
was separately opposed by Benguet taxes thereon.The Bureau of Forestry

Midterm Cases Exclusive Jurisdiction 2


Development also interposed its objection, HELD:
arguing that the landsought to be registered NO. Our holding is that Benguet and Atok
was covered by the Central Cordillera Forest have exclusive rights to the propertyin
Reserve under Proclamation No. 217 dated question by virtue of their respective mining
February 16, 1929. Moreover, by reason of its claims which they validly acquired beforethe
nature, it wasnot subject to alienation under Constitution of 1935 prohibited the alienation
the Constitutions of 1935 and 1973.The trial of all lands of the public domain
court denied the application, holding that the exceptagricultural lands, subject to vested
applicants had failed to prove their claim of rights existing at the time of its adoption. The
possession and ownership of the land sought landwas not and could not have been
to be registered.The applicants appealed to the transferred to the private respondents by
respondent court, which reversed the trial virtue of acquisitive prescription, nor could its
court andrecognized the claims of the use be shared simultaneously by them and
applicant, but subject to the rights of Benguet themining companies for agricultural and
and Atok respecting their mining claims. In mineral purposes. It is true that the
other words, the Court of Appeals affirmed the subject property was considered forest land a
surfacerights of the de la Rosas over the land nd included in the Central Cordillera ForestRe
while at the same time reserving the sub- serve, but this did not impair the rights
surfacerights of Benguet and Atok by virtue already vested in Benguet and Atok at
of their mining claims. Both Benguet and thattime. Such rights were not affected either
Atok haveappealed to this Court, invoking by the stricture in the
their superior right of ownership. CommonwealthConstitution against the
alienation of all lands of the public domain
ISSUE: except thoseagricultural in nature for this was
WHETHER OR NOT APPLICANTS ALL made subject to existing rights. The perfection
SURNAMED DELA ROSA HAVESUPERIOR of themining claim converted the property to
RIGHTS OF OWNERSHIP OVER THE mineral land and under the laws then in
SURFACE RIGHTS OVER THELAND IN forceremoved it from the public domain. By
QUESTION WHILE OPPOSITORS BENGUET such act, the locators acquired exclusive
CONSOLIDATED, INC. rightsover the land, against even the
AND ATOK BIG WEDGE MINING COMPANY government, without need of any further act
ARE RESERVED OF THEIR SUB-SURFACE such as the purchase of the
RIGHTS BY VIRTUE OF THEIR MINING land or the obtention of a patent
CLAIM AS DECIDED BYTHE RESPONDENT over it. As the land had become the private
COURT. property of the locators, they had the right to
transfer the same, as they did, to Benguet and

Midterm Cases Exclusive Jurisdiction 3


Atok. The Court of Appeals justified this by to extract the minerals therein in the exercise
saying there is no conflict of interest of its sovereignprerogative. The land is thus
between the owners of the surface rights and converted to mineral land and may not be
the owners of the sub-surfacerights. This is used by
rather doctrine, for it is a well-known any private party, including the registered ow
principle that the owner of pieceof land has ner thereof, for any other purpose that willimp
rights not only to its surface but also to ede the mining operations to be undertaken
everything underneath and theairspace above therein. The Regalian doctrine then extends
it up to a reasonable height. Under the not only to land but also to all natural wealth
aforesaid ruling, the land isclassified as that may be found in the bowels of the earth.
mineral underneath and agricultural on the
surface, subject to separateclaims of title.
This is also difficult to understand, especially
REPUBLIC OF THE PHILIPPINES, Petitione
in its practical application.The Regalian
r, vs. JEREMIAS AND DAVIDHERBIETO,Res
doctrine which, as its name implies, is
pondents. [G.R. No. 156117. May 26,
intended for the benefit of the State,not of
2005]
private persons. The rule simply reserves to
the State all minerals that may befound in
Facts: This is a petition for review
public and even private land devoted to
assailing the decision of the CA, affirming the
"agricultural, industrial,
decision of theMTC granting the application
commercial,residential or (for) any purpose
for land registration of
other than mining." Thus, if a person is the
the respondents.Respondents filed a single
owner of agricultural land in which minerals
application for two parcel of lands located at
are discovered, his ownership of such land
Cabangahan,Consolacion, Cebu. They claim
does notgive him the right to extract or utilize
to be the owner of said lots by virtue of its
the said minerals without the permission of
purchase fromrespondents parents. They also
theState to which such minerals belong.The
submitted pertinent documents to prove their
flaw in the reasoning of the respondent court
claim and withemphasis on the Certifications
is in supposing that the rights over theland
by the Community Environment and Natural
could be used for both mining and non-mining
Resources Office(CENRO) of the DENR on its
purposes simultaneously. Thecorrect
finding that the Subject Lots are alienable and
interpretation is that once minerals are
disposable, byvirtue of Forestry
discovered in the land, whatever theuse to
Administrative Order No. 4-1063, dated 25
which it is being devoted at the time, such use
June 1963.An initial hearing was set on Sept.
may be discontinued by theState to enable it
3, 1999 and notifications were posted in

Midterm Cases Exclusive Jurisdiction 4


conspicuous places on the subject lots and on theFreeman and the Banat News was only
the municipal hall. The notice was also done 3 months after the hearing which
published in the officialgazette on Aug. 2 1999 rendersinutile the intention of the mandatory
and on the Freeman and Banat news on Dec. publication.
19, 2.) Respondents failed to comply with the
1999.MTC rendered a decision granting title t required period of possession of the
o the respondents. Petitioner assailed saiddeci SubjectLots for the judicial confirmation or
sion on the grounds of: 1.) Jurisdiction, since legalization of imperfect or incomplete title.
there was a procedural defect in the filing of The said lots are public lands classified as
asingle application for two parcels of land; 2.) alienable and disposable only on June 25,
Respondents failed to establish that they and 1963and the respondents were seeking for a
their predecessors-in-interest had been in confirmation of imperfect or incomplete
open, continuous, and adverse possession of titlethrough judicial legalization. Under Sec.48
the SubjectLots in the concept of owners since of the Public Land Act, which is the rulinglaw
12 June 1945 or earlier. in this case, Respondents were not able to
prove their continuous ownership of theland
Issue: since June 12, 1945 or earlier, because said
1.)Does the MTC have the jurisdiction? lands were only classified as alienableand
2.) Did the respondents had open, disposable only on June 25, 1963.
continuous, and adverse possession of the
CRUZ VS SECRETARY OF DENR
Subject Lotsin the concept of owners since 12
June 1945 or earlier.
FACTS: Petitioners Isagani Cruz and
Cesar Europa filed a suit for prohibition and
Held:
mandamus as citizens and taxpayers,
1.) On Jurisdiction
assailing the constitutionality of certain
the procedural defect or the misjoinder,
provisions of Republic Act No. 8371, otherwise
wherein two or more distinctor contradicting
known as the Indigenous Peoples Rights Act
rights or demands are joined, does not remove
of 1997 (IPRA) and its implementing rules and
the courts jurisdiction.HOWEVER, in the case
regulations (IRR). The petitioners assail
at hand there was indeed a lack of jurisdiction
certain provisions of the IPRA and its IRR on
not because of themisjoinder but because of:
the ground that these amount to an unlawful
a.) THE REQUIREMENT FOR PUBLICATION, i
deprivation of the States ownership over
t ismandatory that the publication be made in
lands of the public domain as well as minerals
the official gazette and in a newspaper
and other natural resources therein, in
of general circulation before the initial
violation of the regalian doctrine embodied in
hearing. As we can notice the publication on
section 2, Article XII of the Constitution.

Midterm Cases Exclusive Jurisdiction 5


FACTS: The principal facts admitted by
ISSUE: Do the provisions of IPRA the pleadings may be stated as follows: In
contravene the Constitution? January, 1920, the petitioner Fortunato
Ortua filed an application with the Bureau of
HELD: No, the provisions of IPRA do not Lands for the purchase of a tract of public
contravene the Constitution. Examining the land situated in the municipality of San Jose,
IPRA, there is nothing in the law that grants Province of Camarines Sur. Following an
to the ICCs/IPs ownership over the natural investigation conducted by the Bureau of
resources within their ancestral domain. Lands, Ortuas application was rejected,
Ownership over the natural resources in the allowing him, however, to file a sale or lease
ancestral domains remains with the State and application for the portion of the land
the rights granted by the IPRA to the ICCs/IPs classified to be suitable for commercial
over the natural resources in their ancestral purposes. Two motions for reconsideration of
domains merely gives them, as owners and the decision were filed and denied. On appeal
occupants of the land on which the resources to the then Secretary of Agriculture and
are found, the right to the small scale Natural Resources (Agriculture and
utilization of these resources, and at the same Commerce), the decision was affirmed.
time, a priority in their large scale
It should be explained that one condition for
development and exploitation.
the purchase of a tract of public agricultural
land, provided by the Public Land Law, Act
Additionally, ancestral lands and ancestral
No. 2874, in its sections 23 and 88, is that
domains are not part of the lands of the
the purchaser shall be a citizen of lawful age
public domain. They are private lands and
of the Philippine Islands or of the United
belong to the ICCs/IPs by native title, which is
States. Fortunato Ortua in his application
a concept of private land title that existed
stated that he was a Filipino citizen, but the
irrespective of any royal grant from the State.
Director of Lands held that on the contrary,
However, the right of ownership and
Ortua was a Chinese citizen. The Dir of Land
possession by the ICCs/IPs of their ancestral
established the ff facts: Fortunato Ortua was
domains is a limited form of ownership and
born in 1885 in Lagonoy, Camarines Sur,
does not include the right to alienate the
Philippine Islands, being the natural son of
same.
Irene Demesa, a Filipina, and Joaquin Ortua,
a Chinese. In 1896 Fortunato was sent to
FORTUNATO ORTUA VS. ENCARNACION
China to study. While he was in China his
father and mother were legally married.

Midterm Cases Exclusive Jurisdiction 6


Fortunato returned to the Philippines in 1906, question of law, is in no sense conclusive
that is, when he was twenty-one years of age. upon the courts, but is subject to review. Any
And that even if presumptively Fortunato action of the Director of Lands which is based
Ortua was a Philippine citizen, certain acts of upon a misconstruction of the law can be
Ortua were pointed to as demonstrating that corrected by the courts.
he had forfeited his Philippine citizenship.
The Director of Lands gave too much
ISSUE: WON the question of law arising from prominence, we think, to two minor facts,
the undisputed evidence was correctly decided susceptible of explanation. When Ortua
by the Director of Lands. returned from China at the age of twenty-one,
it was the most natural thing in the world for
HELD: NO.ON QUESTION OF FACT.
him to land as a Chinese, for this would
A decision rendered by the Director of Lands
facilitate entry and obviate complications.
and approved by the Secretary of Agriculture
Again, when Ortua applied for the registration
and Commerce, upon a question of fact is
of a boat, there may have been any number of
conclusive and not subject to be reviewed by
reasons why he did not care to appeal from
the courts, in the absence of a showing that
the decision of the Insular Collector of
such decision was rendered in consequence of
Customs. On the other hand, some
fraud, imposition, or mistake, other than error
consideration should be given to the intention
of judgment in estimating the value or effect of
of the petitioner, and he vigorously insists
evidence, regardless of whatever or not it is
that it is his desire to be considered a
consistent with the preponderance of the
Philippine citizen. He has taken a Filipino
evidence, so long as there is some evidence
name. He has gone into business and has
upon which the finding in question could be
improved the property here in question to a
made.
great extent. There has been no implied
There is, however, another side to the case. It
renunciation of citizenship, because the
certainly was not intended by the legislative
petitioner has been domiciled in these Islands
body to remove from the jurisdiction of courts
except for a short period during his infancy
all right to review decisions of the Bureau of
when he temporarily sojourned in China for
Lands, for to do so would be to attempt
study. On the contrary, he states that he has
something which could not be done legally.
always considered himself to be a Filipino,
Giving force to all possible intendments
and that he has elected to remain as a
regarding the facts as found by the Director of
Philippine citizen. Therefore, on the facts
Lands
found by the Director of Lands, we hold that

ON QUESTION OF LAW. The decision of clear error of law resulted in not considering

the Director of Lands approved by the petitioner a Philippine citizen and so qualified

Secretary of Agriculture and Commerce on a

Midterm Cases Exclusive Jurisdiction 7


under the Public Land Law to purchase public for the caretakers temporary dwelling.
agricultural lands. CA Affirmed RTC: in favor of the
Valerianos
DIRECTOR OF LANDS VS CA
G.R. No. L-58867 June 22, 1984
ISSUES:
Lessons Applicable: Sec. 3 Art. XII 1987
W/N the Courts can reclassify public
Constitution (Land Titles and Deeds)
land - NO
W/N the Valerianos are entitled to
FACTS: Land situated in Obando,
judicial confirmation of title - NO
Bulacan. May 10, 1976: The
Valerianos claimed that they are the co-
HELD: CA reverse
owners in fee simple of the land applied for
NO
partly through: inheritance - 1918;
In effect, what the Courts a quo have
and purchase - May 2, 1958
done is to release the subject property from
the unclassified category, which is beyond
Republic of the Philippines, represented
their competence and jurisdiction
by the Director of the Bureau of Forest
The classification of public lands
Development opposed the application on the
is an exclusive prerogative of the Executive
principal ground that the land applied for is
Department of the Government and not of
within the unclassified region of Obando,
the Courts.
Bulacan, per BF Map LC No. 637 dated
In the absence of such
March 1, 1927; and that areas within the
classification, the land remains as
unclassified region are denominated as forest
unclassified land until it is released
lands and do not form part of the disposable
therefrom and rendered open to disposition.
and alienable portion of the public domain
2. NO
Regalian doctrine: all lands of the
Land was found to be an Unclassified
public domain belong to the State, and that
Region of Obando, Bulacan per BF LC Map
the State is the source of any asserted right
No. 637, certified March 1, 1927. However,
to ownership in land and charged with the
on-the-spot inspection conducted by a
conservation of such patrimony.
representative of this Office, disclosed that
if land is w/in the jurisdiction of the
the same was devoid of any forest growth
Bureau of Forest Development, it would be
and forms part of a well-developed and 100
beyond the jurisdiction of the Cadastral
percent producing fishponds. 2 houses of
Court to register it under the Torrens System
light materials were erected within the area
Since the subject property is still

Midterm Cases Exclusive Jurisdiction 8


unclassified, whatever possession Applicants RULING:
may have had, and, however long, cannot No. the registration of lot no. 2228 is
ripen into private ownership invalid as the lot has already been registered
The conversion of subject under the name of oppositors wife. It was not
property into a fishpond by Applicants, or denied by the appellants that said land was a
the alleged titling of properties around it, public land grant in favor of oppositors wife,
does not automatically render the property because when the Government grants land to
as alienable and disposable. a private individual a patent thereof is
Applicants' remedy lies in the release of recorded and a certificate of title is issued to
the property from its present classification the grantee then it comes within the land
. In fairness to Applicants, and it registration act that after 1 year of issuance
appearing that there are titled lands around said title becomes indefeasible,
the subject property, petitioners-officials incontrovertible and irrevocable. And as the
should give serious consideration to the court ruled in Pamintuan vs. San Agustin that
matter of classification of the land in a cadastral court cannot decree a registration
question. over a land which has already been registered
in an earlier case, and a second decree for
LAHORA VS. DAYANGHIRANG said land is null and void.
37 SCRA 346
FACTS:
On November 26, 1965 appellant REPUBLIC VS CA AND NAGUIT
Francisco Lahora and Toribia Moralizon
petitioned the cpurt for registration of 9 Facts: On January 5, 1993, Naguit filed
parcels of land in Davao. Registration of one of a petition for registration of title of a parcel of
the parcels of land identified as lot no. 2228 land. The application sought a judicial
was opposed by appellee Emilio Dayanghirang confirmation of imperfect title over the land.
who alleged that said lot was already
registered in the name of his wife. Director of The public prosecutor, appearing for the
Lands also filed opposition to the petition government, and Angeles opposed the
arguing that appellants never had sufficient petition. The court issued an order of general
title over such land sought to be registered nor default against the whole world except as to
were they been in open, continuous or Angeles and the government.
notorious possession of said lot.
ISSUE: The evidence revealed that the subject parcel
Whether or not the petition for the of land was originally declared for taxation
original registration of lot no. 2228 is valid? purposes in the name of Urbano in

Midterm Cases Exclusive Jurisdiction 9


1945. Urbano executed a Deed of Quitclaim lands of the public domain which were not
in favor of the heirs of Maming, wherein he declared alienable or disposable before June
renounced all his rights to the subject 12, 1945 would not be susceptible to original
property and confirmed the sale made by his registration, no matter the length of
father to Maming sometime in 1955 or unchallenged possession by the occupant.
1956. Subsequently, the heirs of Maming
executed a deed of absolute sale in favor of
respondent Naguit who thereupon started Issue: Whether or not it is necessary
occupying the same. under Section 14(1) of the Property
Registration Decree that the subject land be
Naguit constituted Blanco, Jr. as her attorney- first classified as alienable and disposable
in-fact and administrator. The administrator before the applicants possession under a
introduced improvements, planted trees in bona fide claim of ownership could even start.
addition to existing coconut trees which were
then 50 to 60 years old, and paid the Held: Section 14 of the Property
corresponding taxes due on the subject land. Registration Decree, governing original
registration proceedings, provides:
Naguit and her predecessors-in-interest had
occupied the land openly and in the concept of SECTION 14. Who may apply. The following
owner without any objection from any private persons may file in the proper Court of First
person or even the government until she filed Instance an application for registration of title
her application for registration. to land, whether personally or through their
duly authorized representatives:
The OSG argued that the property which is in
open, continuous and exclusive possession (1) those who by themselves or through their
must first be alienable. Since the subject land predecessors-in-interest have been in open,
was declared alienable only on October 15, continuous, exclusive and notorious
1980, Naguit could not have maintained a possession and occupation of alienable and
bona fide claim of ownership since June 12, disposable lands of the public domain under a
1945, as required by Section 14 of the bona fide claim of ownership since June 12,
Property Registration Decree, since prior to 1945, or earlier.
1980, the land was not alienable or
disposable. (2) Those who have acquired ownership over
private lands by prescription under the
The OSG suggested an interpretation that all provisions of existing laws.

Midterm Cases Exclusive Jurisdiction 10


possession even if in good faith. However, if
There are three obvious requisites for the filing the property has already been classified as
of an application for registration of title under alienable and disposable, as it is in this case,
Section 14(1) that the property in question is then there is already an intention on the part
alienable and disposable land of the public of the State to abdicate its exclusive
domain; that the applicants by themselves or prerogative over the property.
through their predecessors-in-interest have
been in open, continuous, exclusive and In this case, the 3 requisites for the filing of
notorious possession and occupation, and; registration of title under Section 14(1) had
that such possession is under a bona fide been met by Naguit. The parcel of land had
claim of ownership since June 12, 1945 or been declared alienable; Naguit and her
earlier. predecessors-in-interest had been in open,
continuous, exclusive and notorious
The OSG's interpretation would render possession and occupation of the land
paragraph (1) of Section 14 virtually evidenced by the 50 to 60-year old trees at the
inoperative and even precludes the time she purchased the property; as well as
government from giving it effect even as it the tax declarations executed by the original
decides to reclassify public agricultural lands owner Urbano in 1954, which strengthened
as alienable and disposable. The one's bona fide claim of ownership.
unreasonableness of the situation would even
be aggravated considering that before June REPUBLIC VS NAGUIAT
12, 1945, the Philippines was not yet even G.R. NO. 134209; JANUARY 24, 2006
considered an independent state.
FACTS:
The more reasonable interpretation of Section Celestina Naguiat filed an application for
14(1) is that it merely requires the property registration of title to four parcels of land
sought to be registered as already alienable located in Panan, Botolan, Zambales. The
and disposable at the time the application for applicant alleges that she is the owner of the
registration of title is filed. If the State, at the said parcels of land having acquired them by
time the application is made, has not yet purchase from its previous owners and their
deemed it proper to release the property for predecessors-in-interest who have been in
alienation or disposition, the presumption is possession thereof for more than thirty (30)
that the government is still reserving the right years; and that to the best of her knowledge,
to utilize the property; hence, the need to said lots suffer no mortgage or encumbrance
preserve its ownership in the State of whatever kind nor is there any person
irrespective of the length of adverse having any interest, legal or equitable, or in

Midterm Cases Exclusive Jurisdiction 11


possession thereof. "Forest lands" do not have to be on mountains
Petitioner Republic opposed on the ground or in out of the way places. The classification
that neither the applicant nor her is merely descriptive of its legal nature or
predecessors-in interest have been in open, status and does not have to be descriptive of
continuous, exclusive and notorious what the land actually looks like.
possession and occupation of the lands in
question since 12 June 1945 or prior thereto,
considering the fact that she has not
established that the lands in question have LA BUGAL BLAAN TRIBAL ASSOCIATION
been declassified from forest or timber zone to INC., et. al. v. VICTOR O. RAMOS,
alienable and disposable property. G.R. No. 127882, 27 January 2004, En
Banc (Carpio-Morales, J.)
ISSUE:
Did the areas in question cease to have the The constitutional provision allowing the
status of forest or other inalienable lands of President to enter into FTAA is a exception
the public domain? to the rule that participation in the
nations natural resources is reserved
HELD: exclusively to Filipinos. Provision must be
No, the said areas are still classified as forest construed strictly against their enjoyment
land.The issue of whether or not respondent by non-Filipinos.
and her predecessors-in-interest have been in
open, exclusive and continuous possession of FACTS: RA 7942 (The Philippine Mining Act)
the parcels of land in question is of little took effect on April 9, 1995. Before the
moment. For, unclassified land cannot be effectivity of RA 7942, or on March 30, 1995,
acquired by adverse occupation or possession; the President signed a Financial and Technical
occupation thereof in the concept of owner, Assistance Agreement (FTAA) with WMCP, a
however long, cannot ripen into private corporation organized under Philippine laws,
ownership and be registered as title. covering close to 100,000 hectares of land in
A forested area classified as forest land of the South Cotabato, Sultan Kudarat, Davao del
public domain does not lose such Sur and North Cotabato. On August 15, 1995,
classification simply because loggers or the Environment Secretary Victor Ramos
settlers have stripped it of its forest cover. issued DENR Administrative Order 95-23,
Parcels of land classified as forest land may which was later repealed by DENR
actually be covered with grass or planted to Administrative Order 96-40, adopted on
crops by kaingin cultivators or other farmers. December 20, 1996.

Midterm Cases Exclusive Jurisdiction 12


investments.
Petitioners prayed that RA 7942, its
implementing rules, and the FTAA between ISSUES:
the government and WMCP be declared Whether or not the Philippine Mining Act is
unconstitutional on ground that they allow unconstitutional for allowing fully
fully foreign owned corporations like WMCP to foreign-owned corporations to exploit
exploit, explore and develop Philippine mineral the Philippine mineral resources.
resources in contravention of Article XII Whether or not the FTAA between the
Section 2 paragraphs 2 and 4 of the Charter. government and WMCP is a service
contract that permits fully foreign
In January 2001, WMC - a publicly listed owned companies to exploit the
Australian mining and exploration company - Philippine mineral resources.
sold its whole stake in WMCP to Sagittarius HELD:
Mines, 60% of which is owned by Filipinos
while 40% of which is owned by Indophil First Issue: RA 7942 is Unconstitutional
Resources, an Australian company. DENR
approved the transfer and registration of the RA 7942 or the Philippine Mining Act of 1995
FTAA in Sagittarius name but Lepanto is unconstitutional for permitting fully foreign
Consolidated assailed the same. The latter owned corporations to exploit the Philippine
case is still pending before the Court of natural resources.
Appeals.
Article XII Section 2 of the 1987 Constitution
EO 279, issued by former President Aquino on retained the Regalian Doctrine which states
July 25, 1987, authorizes the DENR to accept, that All lands of the public domain, waters,
consider and evaluate proposals from foreign minerals, coal, petroleum, and other minerals,
owned corporations or foreign investors for coal, petroleum, and other mineral oils, all
contracts or agreements involving wither forces of potential energy, fisheries, forests or
technical or financial assistance for large scale timber, wildlife, flora and fauna, and other
exploration, development and utilization of natural resources are owned by the State. The
minerals which upon appropriate same section also states that, the exploration
recommendation of the (DENR) Secretary, the and development and utilization of natural
President may execute with the foreign resources shall be under the full control and
proponent. WMCP likewise contended that the supervision of the State.
annulment of the FTAA would violate a treaty
between the Philippines and Australia which Conspicuously absent in Section 2 is the
provides for the protection of Australian provision in the 1935 and 1973 Constitution

Midterm Cases Exclusive Jurisdiction 13


authorizing the State to grant licenses, construed strictly against their enjoyment by
concessions, or leases for the exploration, non-Filipinos. Therefore, RA 7942 is invalid
exploitation, development, or utilization of insofar as the said act authorizes service
natural resources. By such omission, the contracts. Although the statute employs the
utilization of inalienable lands of the public phrase financial and technical agreements
domain through license, concession or lease is in accordance with the 1987 Constitution, its
no longer allowed under the 1987 pertinent provisions actually treat these
Constitution. agreements as service contracts that grant
beneficial ownership to foreign contractors
Under the concession system, the contrary to the fundamental law.
concessionaire makes a direct equity
investment for the purpose of exploiting a The underlying assumption in the provisions
particular natural resource within a given of the law is that the foreign contractor
area. The concession amounts to complete manages the mineral resources just like the
control by the concessionaire over the foreign contractor in a service contract. By
countrys natural resource, for it is given allowing foreign contractors to manage or
exclusive and plenary rights to exploit a operate all the aspects of the mining
particular resource at the point of extraction. operation, RA 7942 has, in effect, conveyed
beneficial ownership over the nations mineral
The 1987 Constitution, moreover, has deleted resources to these contractors, leaving the
the phrase management or other forms of State with nothing but bare title thereto.
assistance in the 1973 Charter. The present
Constitution now allows only technical and The same provisions, whether by design or
financial assistance. The management and inadvertence, permit a circumvention of the
the operation of the mining activities by constitutionally ordained 60-40%
foreign contractors, the primary feature of the capitalization requirement for corporations or
service contracts was precisely the evil the associations engaged in the exploitation,
drafters of the 1987 Constitution sought to development and utilization of Philippine
avoid. natural resources.

The constitutional provision allowing the When parts of a statute are so mutually
President to enter into FTAAs is an exception dependent and connected as conditions,
to the rule that participation in the nations considerations, inducements or
natural resources is reserved exclusively to compensations for each other as to warrant a
Filipinos. Accordingly, such provision must be belief that the legislature intended them as a

Midterm Cases Exclusive Jurisdiction 14


whole, then if some parts are Constitution. They are precisely the vices that
unconstitutional, all provisions that are thus the fundamental law seeks to avoid, the evils
dependent, conditional or connected, must fail that it aims to suppress. Consequently, the
with them. contract from which they spring must be
struck down.
Under Article XII Section 2 of the 1987
Charter, foreign owned corporations are DE LOS ANGELES v. SANTOS
limited only to merely technical or financial IN THE MATTER OF THE APPLICATION FOR
assistance to the State for large scale REGISTRATION OF LAND. LEONOR DE LOS
exploration, development and utilization of ANGELES, FEDERICO DE LOS ANGELES, ET
minerals, petroleum and other mineral oils. AL., applicants-appellants, vs.
ISIDORO O. SANTOS, ANTONIO ASTUDILLO,
Second Issue: RP Government-WMCP FTAA ET AL., THE DIRECTOR OF LANDS and THE
is a Service Contract PROVINCE OF RIZAL, oppositors-appellees.
G.R. No. L-19615 December 24, 1964;
The FTAA between he WMCP and the Bengzon, JP, J.:
Philippine government is likewise
unconstitutional since the agreement itself is FACTS: On November 21, 1959 an
a service contract. application for registration of title on 12
parcels of land in Ampid, San Mateo, Rizal,
Section 1.3 of the FTAA grants WMCP a fully was filed in the Court of First Instance of Rizal
foreign owned corporation, the exclusive right by Leonor De Los Angeles and 7 co-applicants.
to explore, exploit, utilize and dispose of all They alleged that they are owners pro-
minerals and by-products that may be indiviso and in fee simple of the aforesaid
produced from the contract area. Section 1.2 land.
of the same agreement provides that EMCP The Director of Lands opposed, stating that
shall provide all financing, technology, the land is a portion of the public domain.
management, and personnel necessary for the The Province of Rizal also interposed an
Mining Operations. opposition, asserting the required 3.00
meters strips of public easement on lots
These contractual stipulations and related along Ampid River and the creek.
provisions in the FTAA taken together, grant At the initial hearing, an order of general
WMCP beneficial ownership over natural default was issued except against the Director
resources that properly belong to the State of Lands, the Director of Lands, the Province
and are intended for the benefit of its citizens. of Rizal and 11 private oppositors who
These stipulations are abhorrent to the 1987 appeared therein.

Midterm Cases Exclusive Jurisdiction 15


The private oppositors, Julio HIidalgo one As contended by the applicants, as of Nov. 21,
among them, filed their written opposition 1957 date of application for registration,
claiming that they are the lawful owners of they are already owners pro-indiviso and in
the parcels of land in question, having fee simple. If this is the case, then Julio
acquired homestead patents over said lots. Hidalgos homestead patent over Lot 11 makes
The Land Registration Commissioner in its said lot no longer public.
report stated that (1) Lot 11 was under patent Since proceedings for land registration in rem
No. 95856 in the name of Julio Hidalgo; and (enforcement is upon the property not against
(2) That the land registration case filed by the person), as against a homestead patent
herein applicants-appellants was set for which is NOT, the latter does not therefore
hearing but no decision has as yet been finally dispose of the public or private
received. character of the land. Hence, applicants
The court then ordered the dismissal of the should be given opportunity to prove
case as regards Lot 11 without prejudice on registrable title to Lot 11.
the part of the applicants to pursue the
corresponding remedy in any ordinary action.
Motion for reconsideration having been HEIRS OF MARIO MALABANAN vs.
denied. Hence, this appeal. REPUBLIC OF THE PHILIPPINES
ISSUE: WON a land registration court which GR No. 179987
has validly acquired jurisdiction over a parcel April 29, 2009
of land for registration of title thereto could be
divested of said jurisdiction by FACTS:
a subsequent administrative act consisting in
the issuance by the Director of Lands of a On 20 February 1998, Mario Malabanan filed
homestead patent covering the same parcel of an application for land registration before the
land. RTC of Cavite-Tagaytay, covering a parcel of
HELD: NEGATIVE. It is well-settled that the land situated in Silang Cavite, consisting of
Director of Lands jurisdiction, administrative 71,324 square meters. Malabanan claimed
supervision and executive control extend only that he had purchased the property from
over lands of the public domain and not to Eduardo Velazco, and that he and his
lands already or private ownership. predecessors-in-interest had been in open,
Accordingly, a homestead patent issued by notorious, and continuous adverse and
him over land NOT of the PUBLIC DOMAIN is peaceful possession of the land for more than
a nullity, devoid of force and effect against the thirty (30) years. Velazco testified that the
owner. property was originally belonged to a twenty-

Midterm Cases Exclusive Jurisdiction 16


two hectare property owned by his great- in the manner and for the length of time
grandfather, Lino Velazco. Lino had four sons required by law for confirmation of imperfect
Benedicto, Gregorio, Eduardo and Esteban title. On 23 February 2007, the Court of
the fourth being Aristedess grandfather. Upon Appeals reversed the RTC ruling and
Linos death, his four sons inherited the dismissed the appliocation of Malabanan.
property and divided it among themselves. But
by 1966, Estebans wife, Magdalena, had
become the administrator of all the properties ISSUES:
inherited by the Velazco sons from their
father, Lino. After the death of Esteban and 1. In order that an alienable and disposable
Magdalena, their son Virgilio succeeded them land of the public domain may be registered
in administering the properties, including Lot under Section 14(1) of Presidential Decree No.
9864-A, which originally belonged to his 1529, otherwise known as the Property
uncle, Eduardo Velazco. It was this property Registration Decree, should the land be
that was sold by Eduardo Velazco to classified as alienable and disposable as of
Malabanan. June 12, 1945 or is it sufficient that such
classification occur at any time prior to the
Among the evidence presented by Malabanan filing of the applicant for registration provided
during trial was a Certification dated 11 June that it is established that the applicant has
2001, issued by the Community Environment been in open, continuous, exclusive and
& Natural Resources Office, Department of notorious possession of the land under a bona
Environment and Natural Resources (CENRO- fide claim of ownership since June 12, 1945
DENR), which stated that the subject property or earlier?
was verified to be within the Alienable or
Disposable land per Land Classification Map 2. For purposes of Section 14(2) of the
No. 3013 established under Project No. 20-A Property Registration Decree may a parcel of
and approved as such under FAO 4-1656 on land classified as alienable and disposable be
March 15, 1982. On 3 December 2002, the deemed private land and therefore susceptible
RTC approved the application for registration. to acquisition by prescription in accordance
with the Civil Code?
The Republic interposed an appeal to the
Court of Appeals, arguing that Malabanan had 3. May a parcel of land established as
failed to prove that the property belonged to agricultural in character either because of its
the alienable and disposable land of the public use or because its slope is below that of forest
domain, and that the RTC had erred in finding lands be registrable under Section 14(2) of the
that he had been in possession of the property Property Registration Decree in relation to the

Midterm Cases Exclusive Jurisdiction 17


provisions of the Civil Code on acquisitive
prescription? (b) The right to register granted under Section
48(b) of the Public Land Act is further
4. Are petitioners entitled to the registration of confirmed by Section 14(1) of the Property
the subject land in their names under Section Registration Decree.
14(1) or Section 14(2) of the Property
Registration Decree or both? (2) In complying with Section 14(2) of the
Property Registration Decree, consider that
HELD: under the Civil Code, prescription is
recognized as a mode of acquiring ownership
The Pertition is denied. of patrimonial property. However, public
domain lands become only patrimonial
(1) In connection with Section 14(1) of the property not only with a declaration that these
Property Registration Decree, Section 48(b) of are alienable or disposable. There must also
the Public Land Act recognizes and confirms be an express government manifestation that
that those who by themselves or through the property is already patrimonial or no
their predecessors in interest have been in longer retained for public service or the
open, continuous, exclusive, and notorious development of national wealth, under Article
possession and occupation of alienable and 422 of the Civil Code. And only when the
disposable lands of the public domain, under property has become patrimonial can the
a bona fide claim of acquisition of ownership, prescriptive period for the acquisition of
since June 12, 1945 have acquired property of the public dominion begin to run.
ownership of, and registrable title to, such
lands based on the length and quality of their (a) Patrimonial property is private property of
possession. the government. The person acquires
ownership of patrimonial property by
(a) Since Section 48(b) merely requires prescription under the Civil Code is entitled to
possession since 12 June 1945 and does not secure registration thereof under Section 14(2)
require that the lands should have been of the Property Registration Decree.
alienable and disposable during the entire
period of possession, the possessor is entitled (b) There are two kinds of prescription by
to secure judicial confirmation of his title which patrimonial property may be acquired,
thereto as soon as it is declared alienable and one ordinary and other extraordinary. Under
disposable, subject to the timeframe imposed ordinary acquisitive prescription, a person
by Section 47 of the Public Land Act. acquires ownership of a patrimonial property

Midterm Cases Exclusive Jurisdiction 18


through possession for at least ten (10) years, under Article 420(2) of the Civil Code. Thus, it
in good faith and with just title. Under is insusceptible to acquisition by prescription.
extraordinary acquisitive prescription, a
persons uninterrupted adverse possession of REPUBLIC v. CA and BERNABE
patrimonial property for at least thirty (30) G.R. No. L-40402 March 16, 1987; Paras, J.:
years, regardless of good faith or just title,
ripens into ownership. FACTS: Lot No. 622 of the Mariveles Cadastre
was declared public land in a decision
It is clear that the evidence of petitioners is rendered before the last war in Cadastral Case
insufficient to establish that Malabanan has No. 19, LRC Cadastral Record No. 1097. On
acquired ownership over the subject property July 6, 1965 such lot was segregated from the
under Section 48(b) of the Public Land Act. forest zone and released and certified by the
There is no substantive evidence to establish Bureau of Forestry as an Agricultural Land for
that Malabanan or petitioners as his disposition under the Public Land Act.
predecessors-in-interest have been in On April 26, 1967, Respondents filed in
possession of the property since 12 June 1945 the CFI of Bataan a petition to reopen
or earlier. The earliest that petitioners can Cadastral Case No. 19 to perfect their rights
date back their possession, according to their and register their titles to said lots. They
own evidencethe Tax Declarations they alleged that they acquired ownership and
presented in particularis to the year 1948. possession of said parcels of land by purchase
Thus, they cannot avail themselves of from the original owners thereof, whose
registration under Section 14(1) of the possession of the same including that of the
Property Registration Decree. herein respondents, has always been
continuous, open, active, exclusive, public,
Neither can petitioners properly invoke adverse and in the concept of owners for more
Section 14(2) as basis for registration. While than 30 years.
the subject property was declared as alienable The Director of Forestry filed an
or disposable in 1982, there is no competent opposition to the above petition but later
evidence that is no longer intended for public withdrew the same upon verification of
use service or for the development of the findings that this portion of the timberland
national evidence, conformably with Article had already been released from the mass of
422 of the Civil Code. The classification of the the public forests. Subsequently, the Acting
subject property as alienable and disposable Prov. Fiscal of Bataan, for and in behalf of the
land of the public domain does not change its Director of Lands filed his opposition alleging
status as property of the public dominion that the land is STILL Public Land and as

Midterm Cases Exclusive Jurisdiction 19


such cannot be the subject of a land however long cannot ripen into private
registration proceeding under Act 496. ownership. A parcel of forest land is within the
The lower court adjudicated in favor or exclusive jurisdiction of the Bureau of
respondent Bernabes, finding that the latter Forestry and beyond the power and
have complied with all the terms and jurisdiction of the cadastral court to register
conditions entitling them to a grant. This under the Torrens System.
decision having become final, the Thus, even if the reopening of the
Commissioner of Land Registration issued the cadastral proceedings was at all possible,
corresponding decrees of registration. On the private respondents have not qualified for a
other hand, petitioner DL through the Solicitor grant under Section 48 (b) of CA 141. They
Gen. filed a petition for review of the decrees. can only be credited with 1 year, 9 mos. and
Afterwards, he filed an Amended Petition for 20 days of possession and occupation of the
Review, adding: that respondents executed lots involved, counted from July 6, 1965 when
simulated deeds of sale conveying portions of the lots involved had been segregated from the
the subject parcels to third parties for forest zone and released by the BOF as an
fictitious considerations in order to remove the agricultural land for disposition under the
same from the coverage of Sec. 38 of Act 496, Public Land Act. As such, respondents and
but in truth, buyers are mere dummies of their predecessors in interest could not have
petitioners; hence, not purchasers for value. possessed the lots for the required period of
The CFI denied this petition and on 30 years as disposable agricultural land.
appeal, the CA affirmed the questioned
decision. Petitioners Motion for
Reconsideration having been denied for lack of STA. ROSA REALTY DEVELOPMENT
merit; hence, this petition. CORPORATION V CA
ISSUE: WON THE LOTS CLAIMED BY
RESPONDENTS COULD LEGALLY BE THE Facts:
SUBJECT OF A JUDICIAL CONFIRMATION
OF TITLE UNDER SEC. 48 (b) OF Petitioner Sta. Rosa Realty Development
COMMONWEALTH ACT 141 AS AMENDED BY Corporation was the registered owner of two
R.A. 1942. parcels of land with a total area of 254.6
HELD: NEGATIVE. The Supreme Court ruled hectares. According to petitioner, the parcels
that Sec. 48 (b) of CA 141, as amended, of land are watersheds, which provide clean
applies exclusively to public lands. Forest potable water to the Canlubang community.
lands or areas covered with forests are Petitioner alleged that respondents usurped
excluded. Thus, possession of forest lands, its rights over the property, thereby destroying

Midterm Cases Exclusive Jurisdiction 20


the ecosystem. Sometime in December 1985, ensures an adequate supply of water for
respondents filed a civil case with the Regional future generations and the control of
Trial Court seeking an easement of a right of flashfloods that not only damage property but
way to and from Barangay Casile. By way of cause loss of lives. Protection of watersheds is
counterclaim, however, petitioner sought the an intergenerational responsibility that needs
ejectment of private respondents. After the to be answered now.
filing of the ejectment cases, respondents
petitioned the Department of Agrarian Reform
for the compulsory acquisition of the SRRDC Director Of Lands V. IAC (1986)
property under the CARP. The landholding of
FACTS:
SRRDC was placed under compulsory
Acme Plywood & Veneer Co., Inc., a
acquisition. Petitioner objected to the
corp. represented by Mr. Rodolfo
compulsory acquisition of the property
Nazario, acquired from Mariano and Acer
contending that the area was not appropriate
Infiel, members of the Dumagat tribe
for agricultural purposes. The area was
5 parcels of land
rugged in terrain with slopes of 18% and
possession of the Infiels over the landdates
above and that the occupants of the land were
back before the Philippines was discovered
squatters, who were not entitled to any land
by Magellan
as beneficiaries. The DARAB ruled against the
land sought to be registered is a private
petitioner. On appeal the CA affirmed the
land pursuant to RA 3872 granting
decision of DARAB.
absolute ownership to members of the
non-Christian Tribes on land occupied by
Issue:
them or their ancestral lands, whether
Whether or not the property in question is
with the alienable or disposable public
covered by CARP despite the fact that the
land or within the public domain
entire property formed part of a watershed
Acme Plywood & Veneer Co. Inc., has
area prior to the enactment of R. A. No. 6657
introduced more than P45M worth of
improvements
Held:
ownership and possession of the land
sought to be registered was duly
Watershed is one of those enumerated by
recognized by the government when the
CARP to be exempt from its coverage. We
Municipal Officials of Maconacon, Isabela
cannot ignore the fact that the disputed
donated part of the land as the townsite of
parcels of land form a vital part of an area
Maconacon Isabela
that need to be protected for watershed
IAC affirmed CFI: in favor of
purposes. The protection of watersheds
ISSUES:

Midterm Cases Exclusive Jurisdiction 21


1. W/N the land is already a private land - must also be conceded that Acme had a
YES perfect right to make such acquisition
2. W/N the constitutional prohibition The only limitation then extant was that
against their acquisition by private corporations could not acquire, hold or
corporations or associations applies- NO lease public agricultural lands in excess of
HELD: IAC affirmed Acme Plywood & Veneer 1,024 hectares
Co., Inc
1. YES HEIRS OF MARIA MARASIGAN v. IAC
already acquired, by operation of law not
only a right to a grant, but a grant of the FACTS:
Government, for it is not necessary that a On April 24, 1975, a civil case entitled
certificate of title should be issued in order Maria Marron v. Felicisimo Bazar and Fe
that said grant may be sanctioned by the S. Bazar was filed before the then CFI of
courts, an application therefore is Manila, Br. XIII. This action sought to
sufficient compel defendants Bazar to execute a
it had already ceased to be of the public registrable Deed of Absolute Saleof Lot No.
domain and had become private property, 2-A covered by T.C.T No. 100612 in favor
at least by presumption of Maria Marron.On January 27, 1976,
The application for confirmation is mere while the above case was still pending,
formality, the lack of which does not affect private respondent Marron caused the
the legal sufficiency of the title as would be annotation of a notice of lis pendens at the
evidenced by the patent and the Torrens back of T.C.T. No. 100612.On February 24,
title to be issued upon the strength of said 1976, judgment was rendered in favor of
patent. Maria Marron and the judgment having
The effect of the proof, wherever made, was become final and executory, she filed a
not to confer title, but simply to establish motion for execution which was granted.
it, as already conferred by the decree, if To this, a writ of execution was granted
not by earlier law but theBazars refused to surrender their
2. NO title and to execute the required deed of
If it is accepted-as it must be-that the land sale.On November 29, 1978, the lower
was already private land to which the court ordered the Clerk of Court to execute
Infiels had a legally sufficient and the deed of sale. But uponpresentation of
transferable title on October 29, 1962 the said deed to the Register of Deeds of
when Acme acquired it from said owners, it Manila for registration, the Deputy Clerk of
Court was advisedto secure a court order

Midterm Cases Exclusive Jurisdiction 22


in order to cancel the new title issued in NEGATIVE. The Supreme Court affirmed
favor of one Maria Marasigan. This is due the appellate court, stating that this
to a priordeed of absolute sale in favor of question is resolved in favor of theparty
such person executed on December 18, who had the notice annotated and who
1974. However, it was only on July 5, 1977 won the litigation over the property, Maria
that such deed was registered; hence, Marron in this case.A notice of lis pendens
Marasigans title bears with it the means that a certain property is involved
above notice of lis pendens. The Bazars in a litigation and serves as a notice to the
filed a petition for relief of the February 24 whole worldthat one who buys the same
judgment and while this was pending, the does it at his own risk. It was also a clear
moved to set aside the same on the ground notice to Maria Marasigan that there was a
of lack of jurisdiction over their persons. courtcase affecting her rights to the
Meanwhile, Marrons Land Registration property she had purchased.In the case at
Court case was dismissed by CFI Manila, bar, although Marasigan acquired the
Br. XIII for said court acting as an L.R.C. property in question on December 18,
cannot act under summary proceedings 1974 or a little over four (4) months before
forhaving only limited and special jurisdiction. the filing of Marrons civil action against the
Then, Marron filed another civil case to cancel Bazars, the transaction became effective as
Marasigans TCT. This was denied for being against thirdpersons only on July 5, 1977,
premature. But, on appeal, the IAC ruled when it was registered with the Register of
in favor of Marron by virtue of the notice of Deeds of Manila. It is the act of
lis pendens. Hence, this petition by the registrationwhich creates constructive
heirs of Marasigan, the latter having died notice to the whole world. Section 51 of Act
in the course of the proceedings. 496, as amended by Section 52 of the
PropertyRegistration Decree (P.D. 1529)
ISSUE: provides: Sec. 52. Constructive notice
WON THE PARTY WHO BOUGHT IT WITH upon registration. Every conveyance x x x
A NOTICE OF affectingregistered land shall, if registered,
LIS PENDENS filed or entered in the office of the Register
ANNOTATED AT THE BACK OF HER TITLE of Deeds for the province or city where the
HASTHE BETTER RIGHT TO THE land to
PROPERTY IN QUESTION AS AGAINST which it relates lies, be constructive notice
THE PARTY IN WHOSE FAVOR THE to all persons from the time of
NOTICE WAS MADE. such registering, filing or entering.

HELD:

Midterm Cases Exclusive Jurisdiction 23


MERIDA V. PEOPLE Tansiongco and not by a DENR forest officer.
YES.
FACTS: Petitioner was charged in the RTC of
Romblon with violation of Section 68 of PD 2) W/N petitioner is liable for violation of
705for"cutting, gathering, collecting and Section 68 of PD 705. YES.
removing a lone narra tree inside a private
land over which privatecomplainant Oscar RATIO: 1. The Revised Rules of Criminal
Tansiongco claims ownership. When Procedure list the cases which must be
confronted during the meeting about thefelled initiated by a complaint filed by specified
narra tree, petitioner admitted cutting the tree individuals, non-compliance of which ousts
but claimed that he did so with the permission the trial court of jurisdiction from trying such
ofone Vicar Calix who, according to petitioner, cases. However, these cases concern only
bought the Mayod Property from Tansiongco defamation and other crimes against chastity
in October1987 under a pacto de retro sale. It and not tocases concerning Section 68 of PD
was later found out that he converted the 705. Further, Section 80 of PD 705 does not
narra trunk into lumber.He was found guilty prohibit aninterested person from filing a
by the Trial Court but he appealed to the complaint before any qualified officer for
Court of Appeals reiterating hisdefense of violation of Section 68 of PD705, as
denial. Petitioner also contended that the trial amended.Moreover, here, it was not "forest
court did not acquire jurisdiction over thecase officers or employees of the Bureau of Forest
because it was based on a complaint filed by Development who reported to Hernandez the
Tansiongco and not by a forest officer as tree-cutting in the Mayod Property but
provided under Section 80 of PD 705. CA Tansiongco, aprivate citizen who claims
affirmed the lower courts ruling, but ordered ownership over the Mayod Property. Thus,
the seized lumber confiscated in the Hernandez cannot be faultedfor not
government's favor. Also, it sustained the trial conducting an investigation to determine "if
court's finding that petitioner isbound by his there is prima facie evidence to support
extrajudicial admissions of cutting the narra thecomplaint or report." At any rate,
tree in the Mayod Property without anyDENR Tansiongco was not precluded, either under
permit. Section 80 of PD 705or the Revised Rules,
from filing a complaint before the Provincial

ISSUE: Prosecutor for petitioner's allegedviolation of

1) W/N the trial court acquired Section 68 of PD 705.2) Petitioner is guilt of

jurisdiction over Criminal Case No. 2207 even the second paragraph of section 80, which is

though it wasbased on a complaint filed by the cutting, gathering, collecting,or removing

Midterm Cases Exclusive Jurisdiction 24


of timber from alienable or disposable public of 342,945 square meters GRANTED TCT
land, or from private land without any No. RT-22481 (372302) was issued in 1991.
authority. The court also said that the lumber In 1996, 8 years after the fire the Barques
or processed log is covered by the forest filed a petition with the LRA for administrative
products term in PD 705, as the law does not reconstitution of TCT No. 210177 in the name
distinguish between a raw and processed of Homer Barque also covering Lot 823. In
timber. support of their petition, the Barques
submitted copies of the alleged owners
MANOTOK vs. BARQUE[1] (G.R. Nos. duplicate of the TCT, real estate tax receipts,
162335 & 162605; August 24, 2010; tax declarations and a Plan Fls 3168-D
VILLARAMA, JR., J.) covering the property.
o MANOTOKs opposed alleging that TCT No.
FACTS: 210177 was spurious.
Piedad Estate originally owned by Although both titles of the Manotoks and
Philippine Sugar Estates Development the Barques refer to land belonging to Lot No.
Company, Ltd., La Sociedad Agricola de 823, TCT No. 210177 actually involves 2
Ultramar, the British-Manila Estate Company, parcels with an aggregate area of 342,945
Ltd., and the Recoleto Order of the Philippine square meters, while TCT No. RT-22481
Islands. (It is a Friar Land.) The subject (372302) pertains only to a 1 parcel of land,
parcel Lot No. 823 is part of the Piedad with a similar area of 342,945 square meters.
Estate and is located in QC. 1997 Barques petition was
On 23 December 1903, Piedad Estate was DENIED. Lot. No. 823 already registered in
acquired by the Philippine Government the name of the Manotoks. --> Barques
pursuant to the Friar Lands Act. The MR was denied They appealed to the
certificate of title in the name of the LRA LRA Reversed. LRA found that the
government was OCT No. 614. The Estate was reconstitution of the Manotok title was
placed under the administration of the fraudulent. Hence, it ordered the Barque title
Director of Lands. to be reconstituted. BUT cancellation must
Controversy arising from conflicting claims 1st be sought in a court of competent
over Lot 823 began after a fire gutted portions jurisdiction of the 1991 Manotok TCT.
of the Quezon City Hall on June 11, 1988 The LRA denied the Manotoks MR and the
which destroyed records stored in the Office of Barques prayer for immediate
the Register of Deeds. reconstitution. Both the Manotoks and the
In 1990, Manotoks filed a petition with the Barques appealed the LRA decision to the
LRA for administrative reconstitution of TCT CA. In the CA, Felicitas Manahan filed a
No. 372302 covering Lot No. 823 with an area motion to intervene and sought the dismissal

Midterm Cases Exclusive Jurisdiction 25


of the cases claiming ownership of the subject be. Consequently, the Director of the Legal
property. Division of the LMB recommended to the
2002 and 2003 2 separate divisions of Director of the LMB the reconstituted Manotok
the CA both directed the RD of QC to cancel Title should be reverted to the state.
the Reconstituted Manotok Title and to o Oral arguments were held on July 24, 2007.
reconstitute the Barques valid, genuine and 2008 - En Banc set aside the December
existing TCT No. 210177. Hence, the 2005 1st division decision and entry of
Manotoks filed the present separate petitions judgment recalled and the CAs Amended
which were ordered consolidated on August 2, Decisions in CA-G.R. SP Nos. 66642 and
2004. 66700 were reversed and set aside. The En
December 12, 2005, SC First Banc remanded the case to the CA.
Division affirmed both decisions of the o The CA was directed to receive evidence of
CA. Manotoks filed MR Denied in April and focus on the issue of WON the Manotoks
2006 Resolution. Thereafter, the Manotoks can trace their claim of title to a valid
filed a Motion for Leave to File a Second MR alienation by the Government of Lot No. 823 of
with their MR attached. Denied in June the Piedad Estate, which was a Friar Land.
2006 Resolution. Eventually entry of judgment PURPOSE: to decide WON the title of the
was made in the Book of Entries of Judgment Maotoks should be annulled.
on May 2, 2006. In the meantime, the CAs findings None of the parties were
Barques filed multiple motions with the First able to prove a valid alienation of Lot 823 from
Division for execution of the judgment, while the government in accordance with the
the Manotoks filed an Urgent Motion to Refer provisions of Act No. 1120 otherwise known as
Motion for Possession to the SC En Banc (with the Friar Lands Act. Notably lacking in the
prayer to set motion for oral deed of conveyance of the Manotoks is the
arguments). Case was referred to the En approval of the Secretary of Agriculture and
Banc in July 2006. Commerce as required by Section 18 of the
On September 7, 2006, Felicitas Manahan said law. Upon close scrutiny, the factual
and Rosendo Manahan filed a motion to allegations and voluminous documentary
intervene, to which was attached their petition exhibits relating to the purchase of Lot 823 by
in intervention. They alleged that their the predecessors-in-interest of the claimants
predecessor-in-interest, Valentin Manahan, revealed badges of fraud and irregularity.
was issued Sale Certificate No. 511 covering
Lot No. 823 and attached the findings of the BASIS FOR THEIR CLAIMS FOR OWNERSHIP:
NBI that the documents of the Manotoks were Manotoks Their grandfather bought Lot 823
not as old as they were purported to from the Government in 1919. They have

Midterm Cases Exclusive Jurisdiction 26


since occupied the land, built their houses Conveyance No. 29204 dated December 7,
and buildings on it. The subject land is now 1932, which likewise lacks the approval of the
known as Manotok Compound. Secretary of Agriculture and Natural
Barques Teresita claims her father (Homer) Resources as it was signed only by the
bought land from Emiliano Setosta who had a Director of Lands.
TCT in his name.
Manahans The lot originally belonged to his Act No. 1120 SECTION 18. No lease or sale
parents but was subsequently bought by his made by Chief of the Bureau of Public Lands
wife. They had a caretaker on the property but under the provisions of this Act shall be valid
she was ousted by armed men in 1950s so until approved by the Secretary of the Interior.
they just declared the property for taxation to
protect their rights. It is clear from the foregoing provision and
from jurisprudence that the sale of friar lands
ISSUE: Who has the better right over Lot shall be valid only if approved by the Secretary
No. 823? NO ONE! It belongs to the of the Interior (later the Secretary of
National Government. Agriculture and Commerce).

RATIO: In their Memorandum, the Manotoks


From the proceedings in the CA, it was pointed out that their photocopy of the
established that while records of the DENR- original Deed of Conveyance No. 29204,
LMB indicate the original claimant/applicant sourced from the National Archives, shows on
of Lot 823 as a certain Valentin Manahan, the second page a poorly imprinted
only the Manotoks were able to produce a sale typewritten name over the words Secretary of
certificate in the name of their predecessors- Agriculture and Natural Resources, which
in-interest, certified by the LMB Records name is illegible, and above it an even more
Management Division. In addition, the poorly imprinted impression of what may be a
Manotoks submitted photocopies of original stamp of the Secretarys approval.
documents entitled Assignment of Sale The Manotoks are invoking the
Certificate dated 1919, 1920 and 1923. presumption of regularity in the performance
Sale Certificate No. 1054 was not signed by of the RDs task in issuing the TCT in the
the Director of Lands nor approved by the Manotok name. The Manotoks contend that
Secretary of the Interior. The Certificates of we can assume that the Manotok deed of
Assignment of Sale contained only the conveyance was in fact approved by the
signature of the Director of Lands. The Department Secretary because the register of
Manotoks belatedly secured from the National deeds did issue TCT No. 22813 in the name of
Archives a certified copy of Deed of the buyer Severino Manotok. FURTHER, the

Midterm Cases Exclusive Jurisdiction 27


Manotoks assert that even if we were to ignore Memo provided that the applicant really paid
the presumption of validity in the performance the purchase price and complied with all the
of official duty, Department Memorandum requirements under the Friar Lands Act.
Order No. 16-05 issued on October 27, 2005 o The CA opined that the Manotoks cannot
by then DENR Secretary Michael T. Defensor, benefit from the above department issuance
supplies the omission of approval by the because it makes reference only to those
Secretary of Agriculture and Natural deeds of conveyance on file with the records of
Resources in deeds of conveyances over friar the DENR field offices. The Manotoks copy of
lands. the alleged Deed of Conveyance No. 29204
o NO! These arguments fail. issued in 1932, was sourced from the National
Citing Alonso v. Cebu Country Club which Archives.
applied the rule in the Solid State and Liao Manotoks also point out that the Friar
Cases the absence of approval by the Lands Act itself states that the Government
Secretary of Agriculture and Commerce in the ceases reservation of its title once the buyer
sale certificate and assignment of sale had fully paid the price. (They were claiming
certificate made the sale null and void ab that they fully paid!) Their basis is SECTION
initio. Necessarily, there can be no valid titles 15[2] of the Friar Lands Act.
issued on the basis of such sale or Court found that the old rule would
assignment. support the Manotoks contention however, the
o SC in the MR of the Alonso case underscored new rule Pugeda v. Trias, the conveyance
that the approval is a MADATORY executed in favor of a buyer or purchaser, or
requirement. Approval of the Secretary of the the so-called certificate of sale, is a
Interior cannot simply be presumed or conveyance of the ownership of the property,
inferred from certain acts since the law is subject only to the resolutory condition that
explicit in its mandate. Petitioners have not the sale may be cancelled if the price agreed
offered any cogent reason that would justify a upon is not paid for in full.
deviation from this rule. Clearly, it is the execution of the contract
to sell and delivery of the certificate of sale
DENR Memorandum Order No. 16, that vests title and ownership to the
invoked by both the Manotoks and the purchaser of friar land. Such certificate of sale
Manahans, states that some Deeds of must, of course, be signed by the Secretary of
Conveyance on record in the field offices of the Agriculture and Natural Resources, as evident
LMB do not bear the Secretarys signature from Sections 11[3], 12[4] and the 2nd
despite full payment for the Friar Land. They paragraph of Section 15[5], in relation to
are deemed signed or otherwise ratified by this Section 18.

Midterm Cases Exclusive Jurisdiction 28


Sale Certificate No. 511 allegedly executed by
CONCLUSIONS Valentin Manahan in favor of Hilaria de
Manotoks could not have acquired Guzman, there is no competent evidence to
ownership of the subject lot as they had no show that the claimant Valentin Manahan or
valid certificate of sale issued to them by the his successors-in-interest actually occupied
Government because their Certificate lacks Lot 823, declared the land for tax purposes, or
the signature of the Director of Lands and the paid the taxes due thereon.
Secretary of Agriculture and Natural Even assuming arguendo the existence
Resources and validity of the alleged Sale Certificate No.
The decades-long occupation by the 511 and Assignment of Sale Certificate No.
Manotoks of Lot 823, their payment of real 511 presented by the Manahans, the CA
property taxes and construction of buildings, correctly observed that the claim had become
are of no moment. It must be noted that the stale after the lapse of 86 years from the
Manotoks miserably failed to prove the date of its alleged issuance. Citing Liao v. CA
existence of the title allegedly issued in the the certificates of sale x x x became stale after
name of Severino Mantotok after the latter 10 years from its issuance and hence cannot
had paid in full the purchase price. The be the source documents for issuance of title
Manotoks did not offer any explanation as to more than 70 years later.
why the only copy of TCT No. 22813 was torn
in half and no record of documents leading to Jacinto v. Director of Lands
its issuance can be found in the registry of
deeds. As to the certification issued by the In the case of Jacinto vs. Director of Lands
Register of Deeds of Caloocan, it simply (1926) 49 Phil. 853, the Supreme Court held
described the copy presented as that the acused friar lands, to which the
DILAPIDATED without stating if the original government of the Philippines holds title, are
copy of TCT No. 22813 actually existed in not public lands but private or patrimonial
their records, nor any information on the year property of the government.
of issuance and name of registered owner.
o As we stressed in Alonso: Prescription can En Banc, Ostrand (J): 7 concur

never lie against the Government.


Facts: During the period from 1911 to 1913,
sales certificates were issued by the Bureau of
RE: MANAHANS No copy of the alleged
Lands to Frank W. Carpenter for more than
Sale Certificate No. 511 can be found in the
100 lots of the Tala and Piedad Friar Lands
records of either the DENR-NCR, LMB or
states located in Novaliches, Caloocan, Rizal
National Archives. Although the OSG
including the lots 670, 690, 691, 695, 696,
submitted a certified copy of Assignment of
697 698, 699, 700, 701, 950, 951, 952, 953,

Midterm Cases Exclusive Jurisdiction 29


954, 955, 956, 957, and 1050. The total area construction of permanent improvements
of the land covered by the sales certificates thereon. Copies of the complaint as well as of
being over 1,490 hectares and the purchase the order of 31 March 1925, were filed with
price amounting to about P56,600, of which the register of deeds of the Province of Rizal on
amount Carpenter up to the year 1923, had 11 February 1926, to be recorded as notices of
paid in installments the sum of P16,272. lis pendens. The lots enumerated above were
Under a judgment rendered against Carpenter included in the land sought to be expropriated
in the CFI of Manila (Civil Case 24607), and Nicanor Jacinto was made a party
execution was levied upon all of his right, title defendant in the proceedings. He admitted the
and interest in the lots purchased together existence of the right of condemnation and the
with the improvements thereon, and on 16 necessity for the expropriation, but demanded
November 1923, the sheriff of Rizal sold the the sum of P64,839.33 as indemnity for the
property to Nicanor Jacinto. The sheriffs sale expropriation. As the actual purchase price to
was registered in the Bureau of Lands, be paid by the purchaser from the
assignments of the Bureau of Lands sales Government only amounts to P13,725,
certificates were duly recorded, and including interest, the Metropolitan Water
certificates of assignment were issued and District considered Jacintos demand
delivered to Nicanor Jacinto in September excessive and declined to pay the claim.
1924.
In the month of July 1926, the applicant
On 31 March 1925, the Metropolitan Water
tendered payment to the Director of Lands of
District instituted proceedings in the CFI Rizal
the sum of P4,650 to cover the remaining
for the condemnation of certain parcels of land
balance of the sales price of the lots in
situated in the municipality of Caloocan for
question and demanded a corresponding deed
the construction of an earth dam and a first-
of conveyance for said lots. The Director of
class highway 3 kilometers long, in connection
Lands, upon the advice of the Attorney-
with the so-called Angat Water Works Project,
General, rejected the tender and refused to
and on the same date the CFI Rizal issued an
execute and deliver the instrument of
order authorizing the Metropolitan Water
conveyance demanded from him. Applicant
District to take possession of said parcels of
filed a petition for a writ of mandamus to
land upon deposit with the provincial
compel the Director of Lands to execute a deed
treasurer of the sum of P3,000 as the
of conveyance in favor of the applicant for the
provisional value, fixed by the court, of the
lots enumerated belonging to the Tala Friar
parcels so to be condemned. By virtue of this
Lands Estate in Novaliches.
order, the Metropolitan Water District entered
into occupation of the land and began the

Midterm Cases Exclusive Jurisdiction 30


The Supreme Court ordered the Director of property of the Philippine Government and we
Lands to receive the balance of the purchase can find no law specially enjoining upon the
money for any or all of the lots in question if Director of Lands the duty to execute deeds of
and when payment thereof is tendered by conveyance to purchasers of such lands; on
Jacinto, and denied the petition as to the the contrary, that duty, under section 567 of
execution of deeds of conveyance; without the
costs. Administrative Code, appears to devolve upon
the Governor-General.
1. Proprietary rights, except that of
occupation, not affected by condemnation 4. Director of Land has duty to receive
proceedings purchase money payable under Act 1120
The proprietary rights, except the right of By section 14 of Act No. 1120 the Director of
occupation, are not affected by the Lands is charged with the duty of receiving the
condemnation proceedings until the title has purchase money payable under that Act and
passed to the plaintiff and that does not occur may therefore be compelled by mandamus to
until the award of compensation or damages receive, as a purely ministerial act, such
has been satisfied. purchase money when tendered.

2. Petition for a writ of mandamus not Republic vs. Register of Deeds


proper remedy to compel a conveyance
Mandamus is not the proper remedy to FACTS:
enforce purely contract rights, such as that in
the present case sought to be enforced. (18 R. Petitioner was awarded a 17-hectare parcel of
C. L., 121; Quiogue vs. Romualdez, 46 Phil., land, by virtue of which he was issued an
337.) OCT.

3. Land is patrimonial property of the


Through an investigation conducted by the
Government; Duty to execute deeds of
Bureau of Lands, it was found that the free
conveyance devolved upon the Governor-
patent acquired by Petitioner was fraudulent.
General
A case for falsification of public documents
The writ cannot issue in the present case
was filed by Petitioner was acquitted of the
unless it appears that the Director of Lands
crime.
unlawfully neglects the performance of an act
which the law specially enjoins as a duty
Subsequently, the Solicitor-General filed a
resulting from an office, trust, or station.
complaint against Petitioner, praying for the

(Section 222, Code of Civil Procedure.) The declaration of nullity of the Free Patent and

land in question is private or patrimonial the OCT.

Midterm Cases Exclusive Jurisdiction 31


Agriculture and Natural Resources.
Petitioner's main contention was that the land Classification of public lands is, thus, an
in question was no longer within the exclusive prerogative of the Executive
unclassified public forest land because by the Department, through the Office of the
approval of his application for Free Patent by President. Courts have no authority to do so.
the Bureau of Lands, the land was already
alienable and disposable public agricultural Thus, in controversies involving the
land. He also claimed that the land was a disposition of public agricultural lands, the
small portion of Lot 5139, an area which had burden of overcoming the presumption of
been declared disposable public land by the state ownership of lands of the public domain
cadastral court. lies upon the private claimant.

ISSUE: In the present case, Petitioner failed to present


clear, positive and absolute evidence to
overcome said presumption and to support his
o Whether or not the land is alienable and claim.
disposable public land
Moreover, the fact the Petitioner acquired a
HELD: title to the land is of no moment,
notwithstanding the indefeasibility of title
Under the Regalian Doctrine, all lands not issued under the Torrens System. The
otherwise clearly appearing to be privately- indefeasibility of a certificate of title cannot be
owned are presumed to belong to the State. invoked by one who procured the same by
Forest lands, like mineral or timber lands means of fraud. Fraud here means actual and
which are public lands, are not subject to extrinsic -- an intentional omission of fact
private ownership unless they under the required by law.
Constitution become private properties. In the
absence of such classification, the land Petitioner committed fraud by his failure to
remains unclassified public land until state that the land sought to be registered still
released therefrom and rendered open to formed part of the unclassified forest lands.
disposition.

The task of administering and disposing lands


of the public domain belongs to the Director of REPUBLIC V. AGUNOY (2005)
Lands, and ultimately the Secretary of FACTS:

Midterm Cases Exclusive Jurisdiction 32


Gregorio Agunoy, Sr. filed an application for provisions of the Internal Revenue Law, Act
Free Patent covering 2 parcels of land in the No. 1189, which, in sub-division (e) of section
Sta. Rosa Cadastre in Nueva Ecija. 9 years 146 expressly repeals so much of paragraph
later, he was issued the free patent by the (h), section 43, of Act No. 82 as authorizes
director of lands. The heirs of Eusebio perez municipalities to exact licenses for cockpits,
caused the annotation of adverse claim on the billiards tables, and so forth. It might be
OCT. added that all the provisions of Act No. 82,
Thus the Register of Deeds of Nueva Ecija authorizing taxes of this nature, fall within the
registered the Free Patent and issued the general repeal of subdivision (a) of section 146
corresponding OCT in the name of Gregorio of Act No. 1189, affecting all laws and
Agunoy, Sr. ordinances whereby taxes of this nature are
Hence this petition for review on certiorari. imposed.lawphil.net
ISSUES:
1. WON the Republic is the real party-in- This argument overlooks the fact that the

interest and thus that Agunoy had municipal council had other concern with

acquired free patent over the subject cockpits and billiards rooms than the mere

lots .(No) licensing of them, being charged by sub-

2. WON the title over the subject lands is division ( j) of section 40 of Act No. 82 with

valid for having been acquired in good the prohibition and closing of them, which is

faith and for value. (Yes) in the nature of a police power regulation
rather than of one revenue. In other words,
Petition denied. CA affirmed although municipal councilors may not now
raise revenue from places of amusement of
THE UNITED STATES vs SARMIENTO
this character, they may, nevertheless, police
(1908)
and control them. Therefore, the councilman

FACTS: still has power over them and the reason of


the law prohibiting his interest in them has
In the Court of First Instance of Bohol, not ceased by reason of the Internal Revenue
Baltasar Sarmiento was convicted if having, as Act. It is plain that it was unnecessary to
a public officer-that is, a councilor of the show that the municipal council of Tagbilaran
municipality of Tagbilaran-been interested in had ever licensed cockpits in order to bring
two cockpits and a billard hall therein, them with the terms of Act No. 663, for, as
contrary to the provisions of section 28 of Act cockpits are therein especially mentioned,
No. 82, as amended by Act No. 663. It is they are not required to be covered by the
claimed by the defense that on the phrase "or other permitted games and
principle cessante ratione legis, cessat et ipsa amusements," whatever may be its technical
lex, this Act is rendered ineffective by the constructions. A conviction for a similar

Midterm Cases Exclusive Jurisdiction 33


offense made under the same Act was the aforecited cases is applicable regardless of
sustained by us in the case of the United whether the redemptioner is a coowner or a
States vs. Gray (8 Phil. Rep., 506).AFFIRMED. mortgagor. Public policy favors redemption
regardless of whether the redemptioner is a
HOMENA VS. CASA coowner or mortgagor, although perhaps with
unequal force and effect since each is given a
Doctine: fixed but different period. A coowner desirous
Basically, the plaintiffs supposed cause of of exercising his right of legal redemption is
action rests upon the deed of sale executed by given a period of thirty (30) days from notice of
defendants in their favor on June 15, 1962 the sale within which to avail of the right to
wherein the latter sold a two-hectare portion redeem. Under the free patent or homestead
of the homestead which they were applying for provisions of the Public Land Act a period of
to the plaintiffs on the understanding that the five (5) years from the date of conveyance is
actual conveyance of the said portion to provided, the fiveyear period to be reckoned
plaintiffs would be made only after the lapse of from the date of the sale and not from the date
the five-year period during which, under the of registration in the office of the Register of
Public Land Act, the homestead owner was Deeds. The redemption of extrajudicially
prohibited from transferring his rights. The foreclosed properties, on the other hand, is
agreement is clearly illegal and void ab initio exercisable within one (1) year from the date of
it is intended to circumvent and violate the the auction sale as provided for in Act No.
law. As parties to a void contract, the plaintiffs 3135
have no rights which they can enforce and the SUSI vs RAZON (1925)
court cannot lend itself to its enforcement. FACTS:
LEE CHUY REALTY CORPORATION VS. A complaint filed by Valentin Susi against
COURT OF APPEALS (1995) Angela Razon and the Director of Lands,
Doctine: praying for judgment: (a) Declaring plaintiff
The doctrine in Tolentino, Tioseco and the sole and absolute owner of the parcel of
Belisario cases was jettisoned by the Court of land; (b) annulling the sale made by the
Appeals on the ground that they do not Director of Lands in favor of Angela Razon, on
involve legal redemption by a coowner but by the ground that the land is a private property;
a mortgagor. It concluded that the application (c) ordering the cancellation of the certificate
of the rules on legal redemption by a coowner of title issued to said Angela Razon.
differs from the legal redemption by a ISSUE:
mortgagor. But the law does not distinguish; Who is then the rightful owner of the land?
neither should we. For sure, the principle in Held:

Midterm Cases Exclusive Jurisdiction 34


SC in their decision favoured Valentin Susi.
According to SC there is, the presumption FACTS:
juris et de jure established in paragraph (b) of
section 45 of Act No. 2874, amending Act No. Subject Land-41,296 square meters situated in

926, that all the necessary requirements for a the barrio of Lucnab, Itogon, Benguet.

grant by the Government were complied with,


Parties:
for he has been in actual and physical
possession, personally and through his A. ATOK BIG-WEDGE MINING COMPANY
predecessors, of an agricultural land of the (claiming that the said parcel of land is a
public domain openly, continuously, mineral land.)
exclusively and publicly since July 26, 1894,
with a right to a certificate of title to said land B. TUKTUKAN SAINGAN (claiming that the
under the provisions of Chapter VIII of said said parcel of land is agricultural.)
Act. So that when Angela Razon applied for
Contentions:
the grant in her favor, Valentin Susi had
already acquired, by operation of law, not only
Atok- they contended that the said
a right to a grant, but a grant of the
parcel of land was being registered in the
Government, for it is not necessary that
office of Mining Recorder in 1921 and 1931
certificate of title should be issued in order
pursuant to Philippine Bill of 1902. It is about
that said grant may be sanctioned by the
sixteen years before TUKTUKAN declared the
courts, an application therefore is sufficient,
land in question for taxation purposes and
under the provisions of section 47 of Act No.
thirty four (34) years before private respondent
2874. If by a legal fiction, Valentin Susi had
filed the land registration proceedings in 1965.
acquired the land in question by a grant of the
They also showed the payment of annual
State, it had already ceased to be the public
assessment fees for the said land since 1931.
domain and had become private property, at
least by presumption, of Valentin Susi, Tuktukan- who was 70 years old at the time
beyond the control of the Director of Lands. he testified shows that he acquired the land
Consequently, in selling the land in question from his father-in-law, Dongail, when he
to Angela Razon, the Director of Lands married his daughter; that he was then 18
disposed of a land over which he had no years old; that at the time of his acquisition, it
longer any title or control, and the sale thus was planted with camotes, casava, langka,
made was void and of no effect, and Angela gabi, coffee and avocados; that he lived on the
Razon did not thereby acquire any right. land since his marriage up to the present; that
he has been paying the taxes during the
ATOK BIG-WEDGE MINING COMPANY vs.
Japanese occupation and even before it; that
INTERMEDIATE APPELLATE COURT.

Midterm Cases Exclusive Jurisdiction 35


he was never disturbed in his possession. owner, continuously, open and uninterrupted
Supporting his oral testimony, applicant for a period of more than 30 years.
[Tuktukan] submitted tax declarations x x x
both dated March 20, 1948, the former for a B. He had improve almost 90% of the said

rural land and the latter for urban land and parcel of land.

improvement therein.
C. He had paid tax declaration of the said land

ISSUE: since 1948 up to present.

Whether or not the said parcel of land is a It is evident that Atok had registered the land

mineral land or an agricultural land. prior than Tuktukan but still the SC ruled in
favor of Tuktukan for the reasons:
History of mining Act
A. Payment of annual assessment fee is not
1. Spanish Mining Law of 1867 enough proof. There must be an annual
performance of labor or undertaking of
2. Philippine Bill Of 1902 (American time) improvements in the mine.
when the subject land had been registered
B. When an ocular survey was made, it was
3. Commonwealth Act No. 137 (under the evident that there was No improvements being
1935 Constitution) made in the said land and there is any sign of
mining had happened in the land. Hence, the
4. Executive Order 141 (Pres. Marcos 1968)
petition is DENIED.

5. President Decree No. 1214 (1977)


LA BUGAL-BLAAN TRIBAL ASSOCIATION

All of the mining acts have a common VS RAMOS 445 SCRA 1 (2004)

provision which is the annual performance of FACTS:

labor or undertaking of improvements on the In 1987, President Aquino issued E.O 2796

mine. authorizing the DENR Secretary to accept,


consider and evaluate proposals from foreign
HELD: owned corporations or foreign investors for
contracts or agreements involving either
The SC ruled in favor of the Tuktukan in the
technical or financial assistance for large-scale
reasons that:
exploration, development, and utilization of
minerals, which upon appropriate
A. Tuktukan have proven that he had in
recommendation of the Secretary the
possession of the said land in a concept of an
President may execute with the foreign

Midterm Cases Exclusive Jurisdiction 36


proponent. In 1995, Pres. Ramos approved RA The present case was a consolidation of three
7942 which defines the modes of mineral cases. All three cases originated from an
agreements for mining operations outlines the application for the registration of 9 parcels of
procedure for their filing and approval , land on 1965 located as Benguet Province
assignment, transfer and withdrawal, and applied by De la Rosa. The application was
fixes their terms .Similar provisions govern opposed by Benguet Consolidated Inc., Atok
financial or technical assistance agreements. Big-wedge Mining Company and, Republic
While in 1997, counsels for Petitioners sent a through the Director of Forest Development.
letter to the DENR Secretary demanding to Here are the arguments of the parties:
stop the implementation of R.A 7942 and DAO De la Rosa According to respondents, they
(no.96-40), giving DENR fifteen days from acquired the lots 1-5 from one Balbalio and
receipt to act thereon. Petitioners claim that lots 6-9 from Jaime Alberto on 1964. During
the DENR Secretary acted with or in excess of the trial Balbalio and Alberto claimed that
jurisdiction. they and their predecessors-in-interest were
ISSUE: already in open, continuous, and exlusive
Whether or not R.A 7942 valid? possession of the parcels of land for a long
HELD: time and presented as evidence tax
The Court finds the provisions of R.A 7942 to declarations and tax receipts dating back from
be violative of Sec.2 Art XII of the 1987 1956.
Constitution namely the Regalian and Benguet Petioner Benguet opposed on the
Filipinization of Natural Resources . When the ground that lots 1-5 or June Bug Mining
parts of the statute re so mutually dependent Claim, to which they assert their right, were
and connected as conditions, considerations, sold to them by the successors-in-interest of
inducements, or compensations for each James Kelly who located the claim in
other, as to warrant a belief that the September 1909 and recorded it on October
legislature intended them as a whole, and that 1909. They have been in actual possession
it all could not be carried into effect, the thereof since their purchase on 1934.
legislature would not pass the residue Atok Atok Big-wedge opposed on the
independently, then, if some parts are premise that portions of lots 1-5 and all lots 6-
Unconstitutional, all the provisions which are 9 were covered by the Emma and Fredia
thus dependent, conditional, or connected mineral claims located and registered by
must not fall with them. Petition is granted. Harisson and Reynolds on 1931 which they
claim to have been purchased by them on
REPUBLIC v COURT OF APPEALS & DELA November 1931and had been occupying since
ROSA (1988) then.
Facts:

Midterm Cases Exclusive Jurisdiction 37


Republic Republic argues that all the land mineral lands, the June Bug claim by
sought to be registered was covered by Central Benguet, and the Emma and Fredia claims of
Cordillera Forest Reserve through Atok, were therefore removed from the public
Proclamation No 219 on year 1929. Moreover, domain and had become private properties. As
by reasons of its nature, it was not subject to cited by the highest tribunal, a valid location
alienation under the 1935 and 1973 of a mining claim segregated the area from the
Constitution. public domain. The moment the locator
The trial court denied the application however discovered a valuable mineral deposit on the
on appeal, CA reversed the ruling of the trial lands located, and perfected his location in
court granting the registration modifying accordance with law, the power of the
however the decision recognizing the sub- Government to deprive him of the exclusive
surface rights of the two mining companies. right to the possession and enjoyment of the
Benguet, Atok, and Republic appealed to the located claim was gone. Citing further, the
High Court. legal effect of a valid location of a mining claim
ISSUES: is not only to segregate the area from the
1. Whether or not the constitutional public domain, but to grant to the locator the
prohibition on non-alienation of lands beneficial ownership of the claim and the right
of public domain other that agricultural to a patent therefor upon compliance with the
land applies to the lands in dispute? terms and conditions prescribed by law.
2. Whether or not the CA erred in granting Where there is a valid location of a mining
surface rights to De la Rosa and sub- claim, the area becomes segregated from the
surface rights to the petitioning mining public domain and the property of the locator.
companies? CA, as upheld by SC, said that it is of no
importance whether Benguet and Atok had
HELD:
secured a patent for as held in the Gold Creek
On the first issue, SC held that indeed the
Mining Corp. Case, for all physical purposes of
areas where the lands were located are forest
ownership, the owner is not required to secure
lands. This should not however affect the
a patent as long as he complies with the
vested rights of Benguet and Atok. SC agreed
provisions of the mining laws; his possessory
with the finding of the appellate court that
right, for all practical purposes of ownership,
having acquired the lots from their
is as good as though secured by patent.
predecessors-in-interest who on their part
The oppositors that having complied with all
acquired valid location of the lots prior to the
the requirements of the mining laws, the
1953 Constitution and before the
claims were removed from the public domain,
constitutional ban on private alienation of
and not even the government of the

Midterm Cases Exclusive Jurisdiction 38


Philippines can take away this right from residential, or for any purpose other than
them. The reason is obvious. Having become mining does not include the ownership of, nor
the private properties of the oppositors, they the right to extract or utilize, the minerals
cannot be deprived thereof without due which may be found on or under the surface.
process of law. This is an application of the Regalian doctrine
Since the lots were already private, it can which, as its name implies, is intended for the
therefore be validly conveyed by the locators to benefit of the State, not of private persons.
Benguet and Atok. These rights were The rule simply reserves to the State all
recognized by the 1935 Constitution. minerals that may be found in public and
On the second issue, SC also agrees with CA even private land devoted to "agricultural,
that the properties being private are now industrial, commercial, residential or (for) any
subject to vicissitudes of ownership, purpose other than mining. The correct
meaning,it can change ownership to the interpretation is that once minerals are
modalities provided by the law like discovered in the land, whatever the use to
abandonment and acquisitive prescription like which it is being devoted at the time, such use
the De las Rosas asserted as an alternative in may be discontinued by the State to enable it
this case.However, it denied the claim of the to extract the minerals therein in the exercise
latter first, because it failed to prove their of its sovereign prerogative. The land is thus
continuouss aand exclusive possession and converted to mineral land and may not be
second, because their claim was premised on used by any private party, including the
the assumption that the lots were agricultural registered owner thereof, for any other
whereas they were not. purpose that will impede the mining
SC held therefore that CA erred in granting operations to be undertaken therein, For the
the surface rights to De la Rosa and loss sustained by such owner, he is of course
subsurface to the mining companies entitled to just compensation under the
reiterating the well-known principle that the Mining Laws or in appropriate expropriation
owner of piece of land has rights not only to proceedings.
its surface but also to everything underneath SOUTHEAST MINDANAO GOLD MINING
and the airspace above it up to a reasonable CORPORATION VS BALITE PORTAL MINING
height. SC believes that the rights over the COOPERATIVE 380 SCRA 145 (2002)
lands were indivisible and that it cannot be FACTS:
half-argricultural and half-mineral at the Case involves a rich tract of mineral land
same time. The Court held that CA situated in the Agusan-Davao-Surigao Forest
misinterpreted the provision of the law that Reserve known as the Diwalwal Gold Rush
The ownership of, and the right to the use of Area, the land has been embroiled in
land for agricultural, industrial, commercial, controversy since the mid 80s due to its gold

Midterm Cases Exclusive Jurisdiction 39


deposits. From then, thousands of people RTC alleging the illegal issuance of OTPs
flocked, to stake their respective claims. In allowed the extraction and hauling of gold per
1988, Marcopper was granted Exploration truck load from SEMs mining claim. RPA
Permit (E.P No.133) which included the hotly dismissed petitioners claim.
contested area. In 1997, Petitioner file before the CA against
In 1991, Congress enacted R.A 7076 PMRB Davao, DENR Secretary and Balite
establishing a Peoples Small-Scale Mining which represent all OTP grantees. It prayed for
Program to be implemented by Secretary of the Nullification of Memorandum Order 97-03
DENR and also to authorized the PMRB to on the ground that the direct state
declare and set aside small-scale mining areas utilization espoused therein would impair its
subject to review by the DENR Secretary and vested right under E.P (no.133).
award mining contracts to small-scale miners ISSUE:
under certain conditions. Then, Sec,Factoran Whether or not the Memorandum Order adopt
issued Dept.Administrative Order (No.60) the direct utilization scheme in resolving the
declaring hectares of the Diwalwal area as dispute over the area?
non-forest land open to small-scale mining. HELD:
Subsequently, a petition for cancellation of No. The challenged Memorandum Order (97-
E.P (no.133) and the admission of a Mineral 03) did not conclusively adopt direct state
Production Sharing Agreement (MPSA) utilization as a policy in resolving the
proposal was filed before the DENR Executive Diwalwal dispute. The terms of the
Director docketed as Mining Case (no.8-8-94) Memorandum clearly indicate that what was
Marcopper while case is pending assigned E.P directed there under was merely a study of
(no.133) to Petitioner SEM which in turn this option and nothing else. Contrary to
applied for an integrated MPSA over the land petitioners contention, it did not grant any
covered by the permit. Mines and management/operating or by profit-sharing
Geosciences Bureau registered the agreement to small-scale miners or to any
application. party, for that matter, has simply instructed
Several entities including respondent Balite the DENR Officials concerned to undertake
Communal Portal Mining Corporation filed studies to determine its feasibility.
their oppositions. In 1997, Provincial Mining Consequently, the petition was premature, the
Regulatory Board of Davao passed a said M.O did not impose any obligation on the
Resolution (no.26) authorizing the issuance of claimants or fix any legal relation whatsoever
Ore Transport Permits (OTPs) to small-scale between and among the parties to the dispute.
miners operating in the Diwalwal area. Then, At this stage, petitioner can show no more
petitioner filed a complaint for damage before than a mere apprehension that the State

Midterm Cases Exclusive Jurisdiction 40


through the DENR, would directly take over obtained by spouses Jose Coloma and
the mines after the studies point to its Eugenia Rumbaoa against Macario filed in the
viability. But until the DENR actually does so RTC of Baguio. The spouses were the
and Petitioners fear turn into reality, no valid purchasers in the auction and the certificate
objection can be entertained against M.O 97- of sale was issued in favor of them.
03 on grounds which are purely speculative In the meantime, an application for lode
and anticipatory. patent covering the Bua Fraction Mineral
Claim was filed with the Bureau of Mines.
COMILANG vs. BUENDIA (1967) Abdon Delenela and his co-heirs filed their
FACTS: opposition to the application. Pending
Nicolas Comilang staked a mining claim application, Delenela filed an action for the
known as the "Bua Fraction Mineral Claim" determination of their rights on the land in the
over a parcel of land in Tuding, Benguet, RTC of Baguio to which the said court
Mountain Province, with an area of 76,809 awarded one-half in undivided share in the
square meters, more or less. He stopped the mineral claim in favor of Marcos and the other
exploration but continue to live in the house half also in undivided share in favor of Abdon
built on a portion of the land with his wife and Delenela and co-heirs. Delenela, with the
other relatives. In 1918, Macario Comilang knowledge and conformity of Marcos
and his relatives also settled on a portion of Comilang, redeemed and bought from the
the land with an area of about one (1) hectare, Coloma spouses, the latter's rights, title,
for residential and agricultural purposes. interest and claim to the 1-1/2 hectares of
Surface rights over the area embraced in the land acquired under the certificate of sale. A
original Bua Fraction Mineral Claim of Nicolas writ of possession was issued in their favor.
Comilang soon became the subject of litigation In a petition for certiorari with PI filed in the
in an action to quiet title filed in RTC of RTC of Baguio, the wife of Marcos questioned
Baguio by the other heirs against Macario the power of municipal court to issue said writ
claiming that they bought the rights and of possession on two grounds (1) that conjugal
interest of Nicolas Comilang in the old mining property had been levied upon and sold in the
claim. execution sale, and her share therein is
The court dismissed both claims of ownership affected; and (2) that there can be no
of petitioner and respondent declaring the severance of surface rights over a mineral
area as a public land, but recognized the claim located under the Philippine Bill of
possession of Macario Comilang over 1 1902, and petitioner argued that the sheriff
hectares which was declared for taxation could not have validly sold the surface rights
purposes but later on levied and sold at a in the execution sale of June 1, 1957. The
public auction to satisfy a money judgment court rendered a decision in said case, holding

Midterm Cases Exclusive Jurisdiction 41


that the writ of possession issued by the certificate of sale was issued in favor of the
respondent Municipal Judge was within his judgment creditors. Interest acquired under
competence and jurisdiction. On appeal, the like certificates of sale alone has been
decision became final. described as more than a lien on the property,
For a second time, a petition for certiorari and more than an equitable estate, an inchoate
mandamus with PIwas instituted by Marcos legal title to the property.
Comilang in the RTC of Baguio City seeking The validity of that sale was questioned when
the annulment of the order granting the alias the Municipal Court ordered the eviction of
writ of possession in favor of Delenela and appellant from the land sold on execution, and
Perez, and again the Court of First Instance of the Supreme Court declared in L-18897 that
Baguio threw out the petition in its order the sale was valid. The sale operated to divest
dated October 22, 1964. Hence the petition. appellant of his rights to the land which
ISSUE: vested in the purchasers at the auction sale.
Whether or not the final certificate of sale The parties herein subsequently litigated their
conveying the land described in Tax rights to the mineral claim in Civil Case No.
Declaration No. 4771 to the purchasers in the 735 of the Court of First Instance of Baguio
execution sale is not a valid disposition of a City, and on the basis of their amicable
portion of the public domain, and specially in agreement (appellant was a party in the case),
view of the subsequent issuance of a mineral the court declared the Bua Mineral Claim co-
lode patent over the Bua Mineral Claim by the ownership property of the parties thereto
Director of Mines (Patent issued on November "except the improvements existing thereon".
7, 1966) whereby full ownership not only of There is no room for doubt, therefore, that the
the minerals therein but also of the surface right to possess or own the surface ground is
ground have been conveyed to the patentee separate and distinct from the mineral rights
thereof, and, therefore, the Municipal Court of over the same land. And when the application
Baguio City may no longer eject them from the for lode patent to the mineral claim was
land. prosecuted in the Bureau of Mines, the said
HELD: application could not have legally included the
No. The 1- hectares portions of the Bua surface ground sold to another in the
Fraction Mineral Claim described in Tax execution sale. Consequently, We have to
Declaration No. 4771 in the name of herein declare that the patent procured thereunder,
appellant was levied upon and sold at public at least with respect to the 1- hectares sold
auction to satisfy the money judgment against in execution pertains only to the mineral right
him in Civil Case No. 1433 of the Municipal and does not include the surface ground of
Court of Baguio City, and the corresponding the land in question.

Midterm Cases Exclusive Jurisdiction 42


BENGUET CONSOLIDATED, INC v of Benguet Consolidated, Inc.'s mineral claims
REPUBLIC (1986) is in violation of law.

FACTS: ISSUE:

On 1958, the Republic of the Philippines filed W/N the power of the government to exercise
with the CFI of Benguet and Baguio a power of imminent domain is barred upon the
complaint for expropriation against ten (10) perfection of the mining claim.
defendants and among them is the Benguet
Consolidated, Inc. The Republic stated that it HELD:

needed the property for the purpose of


The petitioner's arguments have no merit. The
establishing and maintaining a permanent site
filing of expropriation proceedings recognizes
for the PMA, a training institution for officers
the fact that the petitioner's property is no
in the AFP, under the direct authority and
longer part of the public domain. The power of
supervision of the Department of National
eminent domain refers to the power of
Defense. It also averred that it had occupied
government to take private property for public
since 1950, the area covered by the mining
use. If the mineral claims are public, there
claims of the defendants and had already
would be no need to expropriate them. The
installed permanent buildings and other
mineral claims of the petitioner are not being
valuable improvements with no less than
transferred to another mining company or to a
P3,000,000.00 in the belief that the area was
public entity interested in the claims as such.
unoccupied portions of the public domain. The
The land where the mineral claims were
petitioner filed a motion to dismiss on the
located is needed for the Philippine Military
ground that the Republic did not need and
Academy, a public use completely unrelated to
has not occupied the areas covered by the
mining. The fact that the location of a mining
above-mentioned mining claims and neither
claim has been perfected does not bar the
have improvements been made on the said
Government's exercise of its power of eminent
areas and that the area covers ground which
domain. The right of eminent domain covers
is rugged in terrain which could have no use
all forms of private property, tangible or
to the PMA. By way of separate and special
intangible, and includes rights which are
grounds for dismissal, petitioner also alleged
attached to land.
that the authority given by the President of
the Philippines for the expropriation The petitioner next raises a procedural point-
proceedings refers to privately owned mineral whether or not in expropriation proceedings
lands, mining interests, and other private an order of condemnation may be entered by
interests of private and that the expropriation the court before a motion to dismiss is denied.

Midterm Cases Exclusive Jurisdiction 43


The petitioner claims that this cannot be
done.
Under these circumstances, the petitioner is
In the instant case of Nieto, the ruling on the estopped from questioning the proceedings of
motion to dismiss was deferred by the trial condemnation followed by the court. We
court in view of a possible amicable cannot condone the inconsistent positions of
settlement. Moreover, after the trial court the petitioner. (See Republic v. Court of
entered an order of condemnation over the Appeals, 133 SCRA 505). it is very clear from
objection of the petitioner, the court issued an the statements of the petitioner that it had
order to the effect that the trial court." already abandoned its earlier stand on the
propriety of expropriation and that its intent
At the hearing conducted by the Board of shifted to the just compensation to be paid by
Commissioners, the counsel for the petitioner the plaintiff for its condemned properties.
manifested that its motion to dismiss was still
pending in court, and requested that the The second issue centers on the amount of
hearing for the presentation of evidence for the just compensation which should be paid by
petitioner be cancelled. At this point, the respondent to the petitioner for the
negotiations between the government and the condemned properties.
petitioner were still going on.
The petitioner assails the appellate court's
In its original decision, the lower court approval of the Commissioners' Report which
overlooked an award of just compensation for fixed the amount of P7,532.46 as just
the petitioner. This triggered off the filing of compensation for the mineral claims. The
the following motions by the petitioner: (1) petitioner contends that this amount is by any
motion for clarification praying that an order standard ridiculously low and cannot be
be issued clarifying the decision insofar as the considered just and that in fact the
compensation to be paid to the petitioner is commissioners' report was rejected by the trial
concerned; (2) motion for new trial and/or court.
reconsideration on the ground that the court
did not award just compensation for the The P7,532.46 just compensation for the

properties of the petitioner; (3) motion to re- petitioner was based on the findings of the

open case on the ground that the issues Board of Commissioners which conducted an

insofar as the petitioner is concerned have not ocular inspection of the mining claims with

been joined since its motion to dismiss has prior notice to all the parties.

not been resolved; and (4) a second motion for


clarification.

Midterm Cases Exclusive Jurisdiction 44


However, the exploration and/or development As stated earlier, the appellate court based its
work on these claims is not sufficient for findings on the Commissioners' Report. The
making any estimate of the value of these petitioner now assails the approval of the
claims for mining purposes. The property has commissioners' report regarding the
possibilities; but, with the limited work done P7,532.46 just compensation to be paid by the
on these claims, no ore body has as yet been government for its four (4) mining claims.
found. Consequently, the value of these claims
cannot be determined at the present time. While it is true that a court may reject a
Commissioners' Report on the ground that the
The petitioner's mining claims were classified amount allowed is palpably inadequate, it is to
as non-producing unpatented claims. It was be noted that the petitioner herein has not
established that the area of the mineral claims supported its stand that the P7,532.46 just
belonging to the petitioner and included in the compensation for its mining claims is by any
Philippine Military Reservation was 25.1082 standard ridiculously low and cannot be
hectares. Hence, the commissioners arrived at considered just.
the total amount of P7,532.46 (25.1082 x
P300.00) as just compensation to be paid to We are not inclined to reject these findings of

the petitioner for its mining claims. facts of the appellate court in the absence of
any contrary evidence pointed to by the
These findings negate the trial court's petitioner.
observation that the commissioners only took
into consideration the surface value of the Moreover, it is to be noted that unlike the

mineral claims. In fact, the lower court plaintiff and other defendants, the petitioner

affirmed the commissioners' report to the did not file any opposition to the

effect that the petitioner herein is only entitled Commissioners' Report in the lower court.

to the surface value of the mineral claims.


The appellate court, however, should have

"Other claims" include the petitioner's mining provided for the payment of legal interest from

claims. Thus, the trial court computed the the time the government took over the

amount to be paid to the petitioner as just petitioner's mining claims until payment is

compensation on the basis of the surface made by the government. MODIFIED.

value of its mining claims.

We find no reason to disturb the lower court's


8. Miners Association of the Philippines vs.
findings on this matter. The petitioner has not
Factoran
advanced any reason for us to reject such
January 16, 1995
findings.
Facts:

Midterm Cases Exclusive Jurisdiction 45


Former President Corazon Aquino issued
Executive Order Nos 211 and 279 in the Issue:
exercise of her legislative powers. EO No. 211 Whether or not the two Department
prescribes the interim procedures in the Administrative Orders valid.
processing and approval of applications for the
exploration, development and utilization of Ruling:
minerals pursuant to Section 2, Article XII of Yes, AO Nos. 57 and 82 are both
the 1987 Constitution. EO No. 279 authorizes constitutional and valid. This is due to the fact
the DENR Secretary to negotiate and conclude that EO279, in effect, gave the Secretary of
joint-venture, co-production, or production- Natural Resources the authority to conclude
sharing agreements for the exploration, jointventure, co-
development, and utilization of mineral production, or production sharing agreements
resources. for the exploration, development and
The issuance and the impeding utilization of mineral resources. Furthermore,
implementation by the DENR of the constitutionality of these administrative
Administrative Order Nos. 57 which declares orders goes to show that the utilization of
that all existing mining leases or agreements inalienable lands of public domain is not
which were granted after the effectivity of the merely done through license, concession or
1987 Constitutionshall be converted into lease since the options are now also open to
production-sharing agreements within one (1) the State through direct undertaking or by
year from the effectivity of these guidelines. entering into co-production, joint venture, or
and Administrative Order No. 82 which production sharing agreements. Accordingly,
provides that a failure to submit Letter of the State, in the exercise of its police power in
Intent and Mineral Production-Sharing this regard, may not be precluded by the
Agreement within 2 years from the effectivity constitutional restriction on non-impairment
of the Department Administrative Order No. of contract from altering, modifying and
57 shall cause the abandonment of the amending the mining leases or agreements
mining, quarry, and sand and gravel claims, granted under Presidential Decree No. 463, as
after their respective effectivity dates amended, pursuant to Executive Order No.
compelled the Miners Association of the 211. Police Power, being co-extensive with the
Philippines, Inc., an organization composed of necessities of the case and the demands of
mining prospectors and claim owners and public interest; extends to all the vital public
claim holders, to file the instant petition needs. The passage of Executive Order No.
assailing their validity and constitutionality 279 which superseded Executive Order No.
before this Court. 211 provided legal basis for the DENR

Midterm Cases Exclusive Jurisdiction 46


Secretary to carry into effect the mandate of , a domestic corporation which is alleged to be
Article XII, Section 2 of the 1987 Constitution. a 100%-owned subsidiary of MMC.
Subsequently, BMG registered SEMs Mineral
9. Production Sharing Agreement
12 Apex Mining Co., Inc. v. Southeast (MPSA)application and the Deed of
Mindanao Gold Mining Corp. (2006) Assignment. Several oppositions were filed.
The Panel of Arbitrators created by the DENR
Facts:The case involves the Diwalwal Gold upheld the validity of EP 133.
Rush Area (Diwalwal), a rich tract of mineral During the pendency of the case, DENR AO
landlocated inside the Agusan-Davao-Surigao No. 2002-18 was issued declaring
Forest Reserve in Davao del Norte and Davao anemergency situation in the Diwalwal
Oriental. Sincethe early 1980s, Diwalwal has Gold Rush Area and ordering the stoppage
been stormed by conflicts brought about by of all miningoperations therein.
numerous mining claims over it.On March Issues:1.
10, 1986, Marcopper Mining Corporation
(MMC) was granted an Exploration W/N EP 133 and its subsequent transfer
Permit(EP 133) by the Bureau of Mines and to SEM is valid.2.
Geo-Sciences (BMG). A long battle ensued
between Apex andMMC with the latter seeking W/N the DENR Secretary has authority to
the cancellation of the mining claims of Apex issue DAO 66 declaring 729 hectares of the
on the ground that suchmining claims were areascovered by the Agusan-Davao-Surigao
within a forest reservation (Agusan-Davao- Forest Reserve as non-forest lands and open
Surigao Forest Reserve) and thus to small-scale mining purposes.3.
theacquisition on mining rights should have
been through an application for a permit to Who (among petitioners Apex and Balite) has
prospect with theBFD and not through priority right over Diwalwal?Held/Ratio:1.
registration of a DOL with the BMG. When it
reached the SC in 1991, the Courtruled INVALID. One of the terms and
against Apex holding that the area is a forest conditions of EP 133 is: That this
reserve and thus it should have applied for a permit shall be for the
permit to prospect with the BFD.On exclusive use and benefit of the permittee
February 16 1994, or his duly authorized agents
MMC assigned all its rights to EP 133 to and shall be used for mineral exploration
Southeast Mindanao GoldMining purposes only and for no other purpose.
Corporation (SEM) While it may be true that SEM is a100%
subsidiary corporation of MMC, there is no

Midterm Cases Exclusive Jurisdiction 47


showing that the former is the duly Only in cases where the corporatefiction was
authorizedagent of the latter. As such, the used as a shield for fraud, illegality or inequity
assignment is null and void may the veil be pierced andremoved. The
as it directly contravenes the termsand doctrine of piercing the corporate veil cannot
conditions of the grant of EP 133.a. therefore be used as a vehicle tocommit
prohibited acts. The assignment of the permit
The Deed of Assignment was a total abdication in favor of SEM is utilized tocircumvent the
of MMCs rights over the permit. condition of nontransferability of the
It is not amere grant of authority to SEM as exploration permit. To allow SEM to avail itself
agent. b. of this doctrine and to approve the validity of
the assignment is tantamount tosanctioning
Reason for the stipulation. an illegal act which is what the doctrine
Exploration permits are strictly granted to precisely seeks to forestall.e.
entities or
individuals possessing the resources and capa PD 463 requires approval of Secretary of
bility to undertake mining operations. Without DENR.
such acondition, non-qualified entities or Also, PD 463 (Mineral
individuals could circumvent the strict ResourcesDevelopment Decree), which is the
requirementsunder the law by the simple governing law when the assignment was
expediency of acquiring the permit from the executed,explicitly requires that the transfer
original permittee.c. or assignment of mining rights, including the
right toexplore a mining area, must be with
Separate personality. the prior approval of the Secretary of DENR.
The fact that SEM is a 100% subsidiary Such is not present in this case.f.
of MMC does notautomatically make it an
agent of MMC. A corporation is an artificial EP 133 expired by non-renewal.
being invested by lawwith a personality Although EP 133 was extended for 12 months
separate and distinct from persons composing until July 6,1994, MMC never renewed its
it as well as from that of anyother legal entity permit prior and after its expiration.With the
to which it may be related. Absent any clear expiration of EP 133 on July 6, 1994,
proof to the contrary, SEM is aseparate and MMC lost any right to the Diwalwal Gold
distinct entity from MMC.d. RushArea. SEM, on the other hand, has not
acquired any right to the said area because
Doctrine of piercing the corporate veil the transfer of EP 133 in its favor is invalid.
inapplicable. Hence, both MMC and SEM have not acquired

Midterm Cases Exclusive Jurisdiction 48


any vested rightover the area covered by wishes. Theexercise of this prerogative lies
EP 133.2. with the Executive Department over which
courts will notinterfere.
NO. The DENR Secretary has no power to 13.
convert forest reserves into non-forest
reserves. Such power is 14. PICOP Resources vs Base Metal 510
vested with the President. The DENR Secretar SCRA 400
y may only recommend to the Presidentwhich
forest reservations are to be withdrawn from Facts: Central Mindanao Mining and
the coverage thereof. Thus, DAO No. 66 isnull Development Corporation entered into a
and void for having been issued in excess of Mines Operating Agreement with Banahaw
the DENR Secretarys authority.3. Mining and Development Corporation.
Pursuant to the terms of the Agreement,
(Since its been held that neither MMC nor SEM Banahaw Mining filed applications for Mining
has any right over Diwalwal, it is Lease Contracts over the mining claims with
thusnecessary to make a the Bureau of Mines. On April 29, 1988,
determination Banahaw Mining was issued a Mines
of the existing right of the remaining claimants, Temporary Permit authorizing it to extract and
petitioners Apex and Balite, in the dispute.) dispose of precious minerals found within its
mining claims
The issue on who has priority right over
Diwalwal is deemedovertaken by the issuance Since a portion of Banahaw Mining's mining
of Proclamation 297 and DAO No. 2002-18, claims was located in petitioner PICOP's
both beingconstitutionally-sanctioned acts of logging concession in Agusan del Sur,
the Executive Branch Banahaw Mining and petitioner PICOP entered
. Mining operations in the into a Memorandum of Agreement, whereby,
DiwalwalMineral Reservation are now, in mutual recognition of each other's right to
therefore, within the full control of the area concerned, petitioner PICOP allowed
the State through theexecutive Banahaw Mining an access/right of way to its
branch. mining claims
Pursuant to Sec. 5 of RA 7942, the State can
either: (1) directly undertake theexploration, Banahaw Mining thereafter converted its
development and utilization of the area or (2) mining claims to applications for Mineral
opt to award mining operations in themineral Production Sharing Agreements.
reservation to private entities including
petitioners Apex and Balite, if it

Midterm Cases Exclusive Jurisdiction 49


While the MPSA were pending, Banahaw confirmed the timber license granted to PICOP
Mining, on December 18, 1996, decided to and warranted the latter's peaceful and
sell/assign its rights and interests over thirty- adequate possession and enjoyment of its
seven mining claims in favor of private concession areas. It was only given upon the
respondent Base Metals Mineral Resources request of the Board of Investments to
Corporation (Base Metals for brevity). The establish the boundaries of PICOP's timber
transfer included mining claims held by license agreement. The Presidential Warranty
Banahaw Mining in its own right as claim did not convert PICOP's timber license into a
owner, as well as those covered by its mining contract because it did not create any
operating agreement with CMMCI. obligation on the part of the government in
favor of PICOP. Thus, the non-impairment
Upon being informed of the development, clause finds no application.
CMMCI, as claim owner, immediately
approved the assignment made by Banahaw Neither did the Presidential Warranty grant
Mining in favor of private respondent Base PICOP the exclusive possession, occupation
Metals, thereby recognizing private respondent and exploration of the concession areas
Base Metals as the new operator of its claims covered. If that were so, the government would
have effectively surrendered its police power to
On November 18, 1997, petitioner PICOP filed control and supervise the exploration,
with the Mines Geo-Sciences Bureau (MGB), development and utilization of the country's
Caraga Regional Office No. XIII an Adverse natural resources.
Claim and/or Opposition to private
respondent Base Metals' application on the ISSUE: W/N the impairment of contracts
following grounds: apply?

I. THE APPROVAL OF THE APPLICATION AND Held: No. The guaranty is merely a collateral
ISSUANCE OF THE MPSA OF BASE METALS inducement.
WILL VIOLATE THE CONSTITUTIONAL
MANDATE AGAINST IMPAIRMENT OF An examination of the Presidential Warranty
OBLIGATION IN A CONTRACT at once reveals that it simply reassures PICOP
of the government's commitment to uphold
The Court of Appeals upheld the decision of the terms and conditions of its timber license
the MAB, ruling that the Presidential and guarantees PICOP's peaceful and
Warranty of September 25, 1968 issued by adequate possession and enjoyment of the
then President Ferdinand E. Marcos merely areas which are the basic sources of raw

Midterm Cases Exclusive Jurisdiction 50


materials for its wood processing complex. The President for the withdrawal of Block 159 from
warranty covers only the right to cut, collect, the coal reservation and its conversion into a
and remove timber in its concession area, and mineral reservation. The petitioner applied for
does not extend to the utilization of other a mineral prospecting permit over Block 159
resources, such as mineral resources, with the OEA, which the latter granted on 4
occurring within the concession. September 1989. On 18 October 1991,
petitioner submitted to the DENR an
The Presidential Warranty cannot, in any application/proposal for a Mineral
manner, be construed as a contractual Production Sharing Agreement (MPSA) over
undertaking assuring PICOP of exclusive Blocks120, 159 and 160 of the Malangas Coal
possession and enjoyment of its concession Reservation. On 21 February 1992, the
areas. Such an interpretation would result in Officer-In-Charge Regional Technical Director
the complete abdication by the State in favor Dario R. Mioza of the Mines and Geo-
of PICOP of the sovereign power to control and Sciences Developmental Service (MGDS)
supervise the exploration, development and advised the petitioner to amend its application
utilization of the natural resources in the area. for MPSA by excluding Block 159 as the same
is covered by the application of the
15. PNOC-ENERGY DEVELOPMENT respondent. Nevertheless, the petitioner did
CORPORATION vs. VENERACION not exclude Block 159 from its MPSA. Records
Facts: also show that it had not applied for nor was
Respondent applied with the Mines and it able to obtain an Exploration Permit from
Geo-Sciences Development Services, DENR, the BMGS over Block 159.On 13 April 1992,
Region IX, Zamboanga City for a Declaration Presidential Proclamation No.890 was issued,
of Location (DOL) over Block 159 of the which effectively excluded Block159 from the
Malangas Coal Reservation, situated at operation of Proclamation No. 284,and
Barangays Payongan and Kauswagan, declared Block No. 159 as government mineral
Alicia,Zamboanga del Sur. On 18 May 1989, reservation open for disposition to qualified
the Office of the Regional Executive Director mining applicants, pursuant to Executive
(RED) of the DENR informed the respondent Order No. 279. Respondent immediately filed,
that his DOL cannot be registered since Block on 28 May 1992, a protest to the petitioners
159 was part of the Malangas Coal inclusion of Block 159 in its application for
Reservation, as provided under Proclamation MPSA before the RED of the DENR Office in
No. 284, issued by the President on19 July Zamboanga City. After the parties were heard,
1938. With the endorsement of the Office of the RED, in an Order, dated 12 April 1993,
Energy Affairs (OEA) and the DENR Secretary, ruled in favor of the respondent and ordered
the respondent petitioned the Office of the the petitioner to amend its MPSA by excluding

Midterm Cases Exclusive Jurisdiction 51


there from Block 159.On 30 July 1993, Thus, the application for a MPSA over Block
petitioner filed an appeal with the DENR 159, while it was still part of a government
Secretary questioning the Orders issued by reservation other than a mineral reservation,
the RED. While the case was pending, was erroneous and improper and could not
respondent applied for a MPSA. On 31 July have been legally accepted. And, since the
1992, he paid the processing fee for a MPSA records show that only one MPSA was filed
covering Block 159 and was able to comply after the issuance of Proclamation 890 that
with all other requirements of the MPSA of the respondents, the preferential right over
application. The MAB ruled that the petitioner Block 159 was acquired by the respondent.
filed its appeal beyond the five-day Issues:
prescriptive period provided under Whether or not the petitioner acquired
Presidential Decree No.463, which was then preferential right on mining rights over Block
the governing law on the matter. The MAB 159.
also decreed that the respondent had
preferential mining rights over Block 159. It Held:
ruled that the proper procedure with respect
to the mining rights application over Block No, in the instant case, petitioner failed
159 when it was still part of the Malangas to state any compelling reason for not filing its
Coal Reservation required the following: (1) appeal within the mandated period. Instead,
application for prospecting permit with the the records show that after failing to comply
OEA or other office having jurisdiction over with the period within which to file their
said reservation; (2)application for exploration motion for reconsideration on time, they again
permit; (3) application for exclusion of the failed to file their appeal before the Office of
land from such reservation;(4) Presidential the DENR Secretary within the time provided
Declaration on exclusion as recommended by by law. Natural resources, mines: as a general
the Secretary; and (5)application for Lease rule, prospecting and
thereof with priority given to holder of exploration of minerals in a government reser
exploration Permit. The MAB noted that vation is prohibited under Sec.13 of PD 463 -
petitioner did not file for an exploration permit However, the same rule provides an exception
nor applied for the exclusion of Block 159. involving instances when the government
Moreover, petitioner filed a MPSA on18 agency concerned allows it. Section13. Areas
October 1991, or almost six (6) months prior Closed to Mining Location. No prospecting
to the issuance of Proclamation No. 890 and exploration shall be allowed: (a)In
excluding Block 159 from the Malangas military, and other Government reservations
Coal Reservation and allowing its disposition. except when authorized by the proper

Midterm Cases Exclusive Jurisdiction 52


Government agency concerned. Section 8 violation of the condition imposed by the
of Presidential Decree No. 463 reiterates the Board of on Induplex in its Joint Venture
rule and clarifies it further by stating that Agreement with Grefco, Inc, prohibiting
prospecting, exploration and exploitation Induplex from mining perlite ore, through an
of minerals on reserved lands other than operating agreement or any other method;
mineral reservations may be undertaken by that Induplex acquired the majority stocks of
the proper government agency. As an Asaphil and that 95% of Ibalons shares were
exception to this rule, qualified persons may also transferred to Virgilio R. Romero, who is a
undertake the said prospecting, exploration stockholder of Induplex, Asaphil and Ibalon.
and exploitation when the said agencies Tuason claimed that said acts adversely
cannot undertake them. affected, not only his interest as claimowner,
but the governments interest as well.
16
17. Asaphil vs Tuason 488 SCRA 126 Asaphil filed its Answer, praying for the
dismissal of the complaint on the ground that
Facts: On March 24, 1975, respondent the DENR has no jurisdiction over the case.
Vicente Tuason, Jr. entered into a Contract for Induplex filed a Motion to Dismiss the
Sale and Purchase of Perlite Ore with Induplex complaint, also on ground of lack of
wherein Induplex agreed to buy all the perlite jurisdiction. Induplex contended that to fall
ore that may be found and mined in Tuasons within the jurisdiction of the DENR, the
mining claim located in Taysa, Daraga, Albay. controversy should involve a mining property
In exchange, Induplex will assist Tuason in and the contending parties must be
securing and perfecting his right over the claimholders and/or mining operators; and
mining claim. that the dispute in this case involves mineral
product and not a mining property, and the
Thereafter, Tuason executed an Agreement to protagonists are claimholders (Tuason) and a
Operate Mining Claims in favor of petitioner buyer (Induplex). DENR affirmed, but the
Asaphil Construction and Development Mines Adjucation Board reversed, stating that
Corporation. Later, Tuason filed with the the complaint is for the cancellation and
Bureau of Mines, DENR a complaint against revocation of the Agreement to Operate Mining
Asaphil and Induplex for declaration of nullity Claims, which is within the jurisdiction of the
of the said Contracts. Tuason alleged in his DENR under Section 7 of PD 1281. The MAB
complaint that the stockholders of Induplex also found that the acquisition by Induplex of
formed and organized Ibalon Mineral the majority stocks of Asaphil, and Induplexs
Resources, an entity whose purpose is to mine assumption of the mining operation violated
any and all kinds of minerals, that this is in the BOI prohibition.

Midterm Cases Exclusive Jurisdiction 53


imposed by the BOI in its Joint Venture
Issue: Whether or not the DENR has Agreement with Grefco, Inc.. Also, Tuason
jurisdiction over Tuasons complaint for the sought the nullity of the Contract for Sale and
annulment of the Contract for Sale and Purchase of Perlite Ore, based on the same
Purchase of Perlite Ore between Tuason and alleged violation. Obviously, this raises a
Induplex, and the Agreement to Operate judicial question, which is proper for
Mining Claims between Tuason and Asaphil; determination by the regular courts.
and second, WON the MAB erred in
invalidating the Agreement to Operate Mining The DENR is not called upon to exercise its
Claims. technical knowledge or expertise over any
mining operations or dispute; rather, it is
Held: Yes. In several cases on mining being asked to determine the validity of the
disputes, the Court recognized a distinction agreements based on circumstances beyond
between (1) the primary powers granted by the respective rights of the parties under the
pertinent provisions of law to the then two contracts. Thus, the DENR Regional
Secretary of Agriculture and Natural Executive Director was correct in dismissing
Resources (and the bureau directors) of an the complaint for lack of jurisdiction over
executive or administrative nature, such as Tuasons complaint consequently, the MAB
granting of license, permits, lease and committed an error in taking cognizance of the
contracts, or approving, rejecting, reinstating appeal, and in ruling upon the validity of the
or canceling applications, or deciding contracts.
conflicting applications, and (2) controversies
or disagreements of civil or contractual nature 18.
between litigants which are questions of a 19.
judicial nature that may be adjudicated only 20. Republic vs Rosemoor 426 SCRA 517
by the courts of justice.The allegations in
Tuasons complaint do not make out a case for Facts: Petitioner Rosemoor Mining and
a mining dispute or controversy within the Development Corporation (spearheaded by
jurisdiction of the DENR. While the four individuals) after having been granted
Agreement to Operate Mining Claims is a permission to prospect for marble deposits in
mining contract, the ground upon which the the mountains of Biak-na-Bato, San Miguel,
contract is sought to be annulled is not due to Bulacan, succeeded in discovering marble
Asaphils refusal to abide by the terms and deposits of high quality and in commercial
conditions of the agreement, but due to quantities in Mount Mabio which forms part of
Induplexs alleged violation of the condition the Biak-na-Bato mountain range.

Midterm Cases Exclusive Jurisdiction 54


There is no merit in the argument that the
Rosemor thereafter applied with the Bureau proclamation is an ex post facto law. It is
of Mines, now Mines and Geosciences Bureau, settled that an ex post facto law is limited in
for the issuance of the corresponding license its scope only to matters criminal in nature.
to exploit said marble deposits. Proclamation 84, which merely restored the
area excluded from the Biak-na-Bato national
Within that same year, License No. 33 was park by canceling respondents license, is
issued by the Bureau of Mines in favor of the clearly not penal in character.
herein petitioners. It is largely unfortunate
that thereafter, Respondent Ernesto Maceda Also at the time President Aquino issued
cancelled the petitioners license stating that Proclamation No. 84 on March 9, 1987, she
their license had illegally been issued, because was still validly exercising legislative powers
it violated Section 69 of PD 463; and that under the Provisional Constitution of 1986.
there was no more public interest served by Section 1 of Article II of Proclamation No. 3,
the continued existence or renewal of the which promulgated the Provisional
license. The latter reason was confirmed by Constitution, granted her legislative power
the language of Proclamation No. 84. until a legislature is elected and convened
According to this law, public interest would be under a new Constitution. The grant of such
served by reverting the parcel of land that was power is also explicitly recognized and
excluded by Proclamation No. 2204 to the provided for in Section 6 of Article XVII of the
former status of that land as part of the Biak- 1987 Constitution.
na-Bato national park.
21.
Issue: whether or not Presidential Lepanto vs WMC 507 SCRA 315
Proclamation No. 84 is valid.
Facts: Lepanto Consolidated and Tampakan
Held: Yes. Proclamation No. 84 is not a bill of Companies dispute ownership of shares of
attainder; that is, a legislative act which stock at WMCP, Tampakan Companies bought
inflicts punishment without judicial trial." Its such shares of stock through their right of
declaration that QLP No. 33 is a patent nullity first refusal under an agreement denominated
is certainly not a declaration of guilt. Neither as tampakan option agreement. Lepanto,
is the cancellation of the license a punishment getting wind of the Sale and Purchase
within the purview of the constitutional Agreement between WMC and Tampakan
proscription against bills of attainder. Companies, wrote, by letter to the DENR
Secretary about the invalidity of said
agreement and reiterated its request for the

Midterm Cases Exclusive Jurisdiction 55


approval of its acquisition of the disputed 22. Dipidio Earth-Savers Multi-Purpose
shares. The Mines and Geosciences Bureau Association vs Gozun 485 SCRA 586
(MGB) of the DENR accordingly informed the
Tampakan Companies of Lepantos position on Facts: After the EDSA Revolution, Cory swiftly
the matter and required the submission of a rolled out EO 279 w/c empowered DENR to
comment thereto. WMCP and WMC by letters stipulate with foreign companies when it
to the MGB, proffered their side. Several other comes to either technical or financial large
letters or position papers were filed by the scale exploration or mining.
parties with the MGB of the DENR. In addition
thereafter, Lepanto filed before the Makati Nine years later, Ramos signed into law RA
RTC a complaint against herein respondents 7942 or the Philippine Mining Act. In 1994,
WMC, WMCP, for specific performance, Ramos already signed an FTAA with Arimco
annulment of contracts, contractual Mining Co, an Australian company. The FTAA
interference and injunction (Civil Case No. 01- authorized AMC (later CAMC) to explore
087). 37,000 ha of land in Quirino and N. Vizcaya
including Brgy Didipio. After the passage of
Issue: Is Lepanto guilty of forum shopping? the law, DENR rolled out its implementing
RRs. Didipio petitioned to have the law and
Held: Yes, Lepanto is guilty of forum shopping. the RR to be annulled as it is unconstitutional
The power of the MGB to rule on the validity of and it constitutes unlawful taking of property.
the questioned agreements of sale, which was
raised by petitioner before the DENR, is In seeking to nullify Rep. Act No. 7942 and its
inextricably linked to the very nature of such implementing rules DAO 96-40 as
agreements over which the MGB has unconstitutional, petitioners set their sight on
jurisdiction under the law. Unavoidably, there Section 76 of Rep. Act No. 7942 and Section
is identity of reliefs that petitioner seeks from 107 of DAO 96-40 which they claim allow the
both the MGB and the RTC. unlawful and unjust taking of private
Forum shopping exists when both actions property for private purpose in contradiction
involve the same transactions, same essential with Section 9, Article III of the 1987
facts and circumstances and raise identical Constitution mandating that private property
causes of actions, subject matter, and issues. shall not be taken except for public use and
Such elements are evidently present in both the corresponding payment of just
the proceedings before the MGB and before compensation. They assert that public
the trial court. respondent DENR, through the Mining Act
and its Implementing Rules and Regulations,

Midterm Cases Exclusive Jurisdiction 56


cannot, on its own, permit entry into a private
property and allow taking of land without (4) the property must be devoted to
payment of just compensation. public use or otherwise
informally appropriated or
Despite petitioners assertion, public injuriously affected;
respondents argue that Section 76 is not a
taking provision but a valid exercise of the (5) the utilization of the property for
police power and by virtue of which, the state public use must be in such a
may prescribe regulations to promote the way as to oust the owner and
health, morals, peace, education, good order, deprive him of beneficial
safety and general welfare of the people. This enjoyment of the property.
government regulation involves the
adjustment of rights for the public good and In the case at bar, Didipio failed to show that
that this adjustment curtails some potential the law is invalid. There is taking involved but
for the use or economic exploitation of private it is not w/o just compensation. Sec 76 of RA
property. Public respondents concluded that 7942 provides for just compensation as well
to require compensation in all such as section 107 of the DENR RR. To wit,
circumstances would compel the government
to regulate by purchase. Section 76. xxx Provided, that any damage to
the property of the surface owner, occupant,
or concessionaire as a consequence of such
Issue: Whether or not RA 7942 and the DENR operations shall be properly compensated as
RRs are valid. may be provided for in the implementing rules
and regulations.
HELD: The SC that the RRs are indeed valid.
The SC noted the requisites of eminent Section 107. Compensation of the Surface
domain. They are; Owner and Occupant- Any damage done to
the property of the surface owners, occupant,
(107) the expropriator must enter a or concessionaire thereof as a consequence of
private property; the mining operations or as a result of the
construction or installation of the
(2) the entry must be for more than infrastructure mentioned in 104 above shall
a momentary period. be properly and justly compensated.
Further, mining is a public policy and the
(3) the entry must be under warrant government can invoke eminent domain to
or color of legal authority;

Midterm Cases Exclusive Jurisdiction 57


exercise entry, acquisition and use of private of that land as part of the Biak-na-Bato
lands. national park.
Issue:
Whether or not Presidential Proclamation No.
84 is valid.
Republic vs. Rosemoor Held:
Yes. We cannot sustain the argument that
Proclamation No. 84 is a bill of attainder; that
Facts: is, a legislative act which inflicts punishment
Petitioner Rosemoor Mining and Development without judicial trial.Its declaration that QLP
Corporation after having been granted No. 33 is a patent nullity is certainly not
permission to prospect for marble deposits a declaration of guilt. Neither is the cancellation of
in the mountains of Biak-na-Bato, San the license a punishment within the purview of the
Miguel, Bulacan, succeeded in discovering constitutional proscription against bills
marble deposits of high qualityand in of attainder. Too, there is no merit in the
commercial quantities in Mount Mabio which argument that the proclamation is an ex post
forms part of the Biak-na-Bato mountain facto law.It is settled that an ex post facto law
range. The petitioner then applied with the is limited in its scope only to matters criminal
Bureau of Mines, now Mines and Geosciences in nature. Proclamation 84, which merely
Bureau, for the issuance of the corresponding restored the area excluded from the Biak-na-
license to exploit said marble deposits.License Bato national park by canceling respondents
No. 33 was issued by the Bureau of Mines in license, is clearly not penal in character. Also at
favor of the herein petitioners.Shortly the time President Aquino issued
thereafter, Respondent Ernesto Maceda Proclamation No. 84 on March 9, 1987, she
cancelled the petitioners license stating that was still validly exercising legislative powers
their license had illegally been issued, because under the Provisional Constitution of 1986.
it violated Section 69 of PD 463; and Section 1 of Article II of Proclamation No. 3,
that there was no more public interest served which promulgated the Provisional
by the continued existence or renewal of the Constitution, granted her legislative power
license. The latter reason was confirmed by until a legislature is elected and convened
the language of Proclamation No. 84. According to under a new Constitution. The grant of such
this law, public interest would be served by power is also explicitly recognized and
reverting the parcel of land that was excluded provided for in Section 6 of Article XVII of the
by Proclamation No. 2204to the former status 1987 Constitution.

Midterm Cases Exclusive Jurisdiction 58


for purposes of exploration of the mining
claims in Tampakan, South Cotabato. The
Option Agreement, among other things,
provides for the grant of the right of first
refusal to the Tampakan Companies in case
WMC Philippines desires to dispose of its
rights and interests in the mining claims
covering the area subject of the agreement.
Lepanto vs WMC Resources
WMC Resources subsequently divested
Philippine Government and WMC Philippines, itself of its rights and interests in the
the local wholly-owned subsidiary of WMC Columbio FTAA, and on 12 July 2000
Resources International Pty. Ltd. (WMC executed a Sale and Purchase Agreement with
Resources) executed a Financial and Technical petitioner Lepanto over its entire
Assistance Agreement, denominated as the shareholdings in WMC Philippines, subject to
Columbio FTAA No. 02-95-XI (Columbio FTAA) the exercise of the Tampakan Companies
for the purpose of large scale exploration, exercise of their right of first refusal to
development, and commercial exploration of purchase the subject shares. On 28 August
possible mineral resources in an initial 2000, petitioner sought the approval of the 12
contract area of 99,387 hectares located in the July 2000 Agreement from the DENR
provinces of South Cotabato, Sultan Kudarat, Secretary.
Davao del Sur, and North Cotabato in In the interim, on 10 January 2001,
accordance with Executive Order No. 279 and contending that the 12 July Agreement
Department Administrative Order No. 63, between petitioner and WMC Philippines had
Series of 1991. expired due to failure to meet the necessary
The Columbio FTAA is covered in part by 156 preconditions for its validity, WMC Resources
mining claims held under various Mineral and the Tampakan Companies executed
Production Sharing Agreements (MPSA) by another Sale and Purchase Agreement, where
Southcot Mining Corporation, Tampakan Sagittarius Mines, Inc. was designated
Mining Corporation, and Sagittarius Mines, assignee and corporate vehicle which would
Inc. (collectively called the Tampakan acquire the shareholdings and undertake the
Companies), in accordance with the Columbio FTAA activities. On 15 January
Tampakan Option Agreement entered into by 2001, Sagittarius Mines, Inc. increased its
WMC Philippines and the Tampakan authorized capitalization to P250 million.
Companies on 25 April 1991, as amended by Subsequently, WMC Resources and
Amendatory Agreement dated 15 July 1994, Sagittarius Mines, Inc. executed a Deed of

Midterm Cases Exclusive Jurisdiction 59


Absolute Sale of Shares of Stocks on 23 In a Decision dated 23 July 2002, the
January 2001. Office of the President dismissed the petition

After due consideration and evaluation


of the financial and technical qualifications of ISSUE:
Sagittarius Mines, Inc., the DENR Secretary
approved the transfer of the Columbio FTAA WHETHER OR NOT the Philippine
from WMC Philippines to Sagittarius Mines, Mining Act of 1995, particularly Section 40
Inc. in the assailed Order. According to said thereof requiring the approval of the President
Order, pursuant to Section 66 of Department of the assignment or transfer of financial or
Administrative Order No. 96-40, as amended, technical assistance agreements should have
Sagittarius Mines, Inc. meets the qualification a retroactive application to the Columbio
requirements as Contractor-Transferee of FTAA.
FTAA No. 02-95-XI, and that the application
for transfer of said FTAA went thru the
procedure and other requirements set forth HELD:
under the law.
NO. Applying the above-cited law
Aggrieved by the transfer of the retroactively would contradict the established
Columbio FTAA in favor of Sagittarius Mines, legal doctrine that statutes are to be
Inc., petitioner filed a Petition for Review of the construed as having only a prospective
Order of the DENR Secretary with the Office of operation unless the contrary is expressly
the President. Petitioner assails the validity of stated or necessarily implied from the
the 18 December 2001 Order of the Secretary language used in the law.
of the Department of Environment and
Natural Resources (DENR) approving the In the case at bar, there is an absence
application for and the consequent of either an express declaration or an
registration of FTAA No. 02-95-XI from WMC implication in the Philippine Mining Act of
Philippines to Sagittarius Mines, Inc.on the 1995 that the provisions of said law shall be
ground that: 1) it violates the constitutional made to apply retroactively, therefore, any
right of Lepanto to due process; 2) it preempts section of said law must be made to apply only
the resolution of very crucial legal issues prospectively, in view of the rule that a statute
pending with the regular courts; and 3) it ought not to receive a construction making it
blatantly violates Section 40 of the Mining Act. act retroactively, unless the words used are so
clear, strong, and imperative that no other

Midterm Cases Exclusive Jurisdiction 60


meaning can be annexed to them, or unless property and allow taking of land without
the intention of the legislature cannot be payment of just compensation.
otherwise satisfied.
Traversing petitioners assertion, public
respondents argue that Section 76 is not a
Didipio vs Gozun
taking provision but a valid exercise of the
Facts: In 1987, Cory rolled out EO 279
police power and by virtue of which, the state
w/c empowered DENR to stipulate with
may prescribe regulations to promote the
foreign companies when it comes to either
health, morals, peace, education, good order,
technical or financial large scale exploration or
safety and general welfare of the people. This
mining. In 1995, Ramos signed into law RA
government regulation involves the
7942 or the Philippine Mining Act. In 1994,
adjustment of rights for the public good and
Ramos already signed an FTAA with Arimco
that this adjustment curtails some potential
Mining Co, an Australian company. The FTAA
for the use or economic exploitation of private
authorized AMC (later CAMC) to explore
property. Public respondents concluded that
37,000 ha of land in Quirino and N. Vizcaya
to require compensation in all such
including Brgy Didipio. After the passage of
circumstances would compel the government
the law, DENR rolled out its implementing
to regulate by purchase.
RRs. Didipio petitioned to have the law and
ISSUE: Whether or not RA 7942 and the
the RR to be annulled as it is unconstitutional
DENR RRs are valid.
and it constitutes unlawful taking of property.
In seeking to nullify Rep. Act No. 7942 and its HELD: The SC ruled against Didipio. The SC
implementing rules DAO 96-40 as noted the requisites of eminent domain. They
unconstitutional, petitioners set their sight on are;
Section 76 of Rep. Act No. 7942 and Section (1) the expropriator must enter a
107 of DAO 96-40 which they claim allow the private property;
unlawful and unjust taking of private
(2) the entry must be for more than a
property for private purpose in contradiction
momentary period.
with Section 9, Article III of the 1987
Constitution mandating that private property (3) the entry must be under warrant

shall not be taken except for public use and or color of legal authority;

the corresponding payment of just (4) the property must be devoted to


compensation. They assert that public public use or otherwise informally
respondent DENR, through the Mining Act appropriated or injuriously affected;
and its Implementing Rules and Regulations,
(5) the utilization of the property for
cannot, on its own, permit entry into a private
public use must be in such a way as to oust

Midterm Cases Exclusive Jurisdiction 61


the owner and deprive him of beneficial Respondent MMC was issued a temporary
enjoyment of the property. permit to operate a tailings sea disposal
system. In the meantime, the National
In the case at bar, Didipio failed to show that
Pollution Control Commission (NPCC) was
the law is invalid. Indeed there is taking
abolished by EO No. 192 dated June 10,
involved but it is not w/o just compensation.
1987, and its powers and functions were
Sec 76 of RA 7942 provides for just
integrated into the Environmental
compensation as well as section 107 of the
Management Bureau and into the Pollution
DENR RR. To wit,
Adjudication Board (PAB).
Section 76. xxx Provided, that any damage to
the property of the surface owner, occupant, On April 11, 1988, the DENR Secretary, in his
or concessionaire as a consequence of such capacity as Chairman of the PAB, issued an
operations shall be properly compensated as Order directing MMC to "cease and desist from
may be provided for in the implementing rules discharging mine tailings into Calancan Bay."
and regulations. This was appealed by the MMC with the Office

Section 107. Compensation of the Surface of the President (OP).

Owner and Occupant- Any damage done to


In line with the directive from the OP, the
the property of the surface owners, occupant,
Calancan Bay Rehabilitation Project (CBRP)
or concessionaire thereof as a consequence of
was created, and MMC remitted the amount of
the mining operations or as a result of the
P30,000.00 a day, starting from May 13, 1988
construction or installation of the
to the Ecology Trust Fund (ETF) thereof.
infrastructure mentioned in 104 above shall
However, on June 30, 1991, MMC stopped
be properly and justly compensated.
discharging its tailings in the Bay, hence, it
Further, mining is a public policy and the likewise ceased from making further deposits
government can invoke eminent domain to to the ETF.
exercise entry, acquisition and use of private
lands. The PAB sought for the enforcement of the
order issued by the OP, however, the CA acted
on Marcoppers petition and ordered the PAB
to refrain and desist from enforcing aforesaid
Order.

Hence, the instant petition.

Republic vs Marcopper

Midterm Cases Exclusive Jurisdiction 62


ISSUE: for violation of pollution control statutes and
regulations. Contrary to the ruling of the CA,
The Court of Appeals erred in ruling that RA 7942 does not vest quasi-judicial powers
Republic Act No. 7942 repealed the provisions in the Mines Regional Director. The authority
of Republic Act No. 3931, as amended by is vested and remains with the PAB. Neither
Presidential Decree No. 984, with respect to was such authority conferred upon the Panel
the power and function of petitioner Pollution of Arbitrators and the Mines Adjudication
Adjudication Board to issue, renew or deny Board which were created by the said law.
permits for the discharge of the mine tailings. The scope of authority of the Panel of
Arbitrators and the Mines Adjudication Board
HELD:
conferred by RA 7942 clearly exclude

The SC held that the CA erred in ruling that adjudicative responsibility over pollution

the PAB had no authority to issue the Order cases.

from the

The ruling of the Court of Appeals that the


Standard Mineral vs CA
PAB has been divested of authority to act on
pollution-related matters in mining operations Petitioner-Appellant Standard Mineral
is anchored on the provisions of RA 7942 Products, Inc. (SMPI, for short) claims that it
(Philippine Mining Act of 1995). However, is the locator of placer mining claims "Celia
Section 19 of EO 192 vested the PAB with the IV" and "Celia VI" containing limestone in
specific power to adjudicate pollution cases in Kaysipot, Antipolo, Rizal, which were duly
general. Sec. 2, par. (a) of PD 984 defines the registered in the Office of the Mining Recorder
term "pollution" as referring to any alteration of Rizal The aforementioned mining claims
of the physical, chemical and biological cover about fifteen (15) hectares of the one
properties of any water, air and/or land hundred-twenty (120) hectares of land
resources of the Philippines , or any discharge registered in the name of Respondent-
thereto of any liquid, gaseous or solid wastes Appellee, Rufino Deeunhong
as will or is likely to create a harmful
environment. After locating the claims, SMPI applied for a
mining lease from the Bureau of Mines. The
On the other hand, the authority of the mines
Landowners opposed the application on the
regional director is complementary to that of
ground that SMPI had entered their land and
the PAB. While the mines regional director has
filed its mining lease application without their
express administrative and regulatory powers
permission.
over mining operations and installations, it
has no adjudicative powers over complaints

Midterm Cases Exclusive Jurisdiction 63


SMPI brought an action in the Court of First Issue:
Instance of Rizal against Respondents-
Appellees praying that it be granted surface WON SMPI is entitled to such surface rights

rights for mining purposes.


Held:

The Landowners traversed the Complaint, by


SMPI is not entitled to said surface rights as it
averring that SMPI is not entitled to the relief
failed to comply with the requisite of prior
demanded because the prospecting was
written permission by the Landowners before
accomplished without previously securing the
entering the private land in question.
Landowners written permission as surface
owners as required by Section 27 of the Section 27 of the Mining Act explicitly
Mining Act (Commonwealth Act No. 137, as provides:
amended)

"Section 27. Before entering private lands the


Trial Court, finding that the mineral claims
prospector shall first apply in writing for
were not located in accordance with law
written permission of the private owner,
dismissed the complaint and, on the
claimant, or holder thereof, and in case of
counterclaim, sentenced SMPI to pay to
refusal by such private owner, claimant, or
Deeunhong and the Tanjuatcos actual
holder to grant such permission, or in case of
damages in the sum of P50,000.00 each,
disagreement as to the amount of
attorneys fees of P5,000.00 and costs. The
compensation to be paid for such privilege of
Appellate Court 1 affirmed that Decision with
prospecting therein, the amount of such
the sole modification that temperate or
compensation shall be fixed by agreement
moderate damages (not actual damages) of
among the prospector, the Director of the
P25,000.00 each were awarded instead.
Bureau of Mines and the surface owner, and

In another Resolution the Appellate Court in case of their failure to unanimously agree

likewise denied, for being devoid of legal as to the amount of compensation, all

interest, the Petition for Intervention filed by questions at issue shall be determined by the

the Republic of the Philippines through the Court of First Instance of the province in

Solicitor General. which said lands are situated in an action


instituted for the purpose by the prospector,
After SMPI elevated the case to this Court for or his principal: Provided, however, that the
review on Certiorari, the Republic reiterated its prospector, or his principal upon depositing
Petition for Intervention, which we granted in with the court the sum considered jointly by
the Resolution. him and the Director of the Bureau of Mines

Midterm Cases Exclusive Jurisdiction 64


and the court to be Just compensation for the same to be without merit, suffice it to state
damages resulting from such prospecting, that "entering" has to be precede "prospecting"
shall be permitted to enter upon, and locate ; "prospecting" necessarily precedes
the said land without such written permission "discovery" ; and a valid "discovery" is
pending final adjudication of the amount of essential for the "location" of a mining claim.
such compensation; and in such case the
prospector, or his principal, shall have a prior "Section 26 of the Mining Act provides that

right as against the world, from the date of his prospecting shall be carried on in accordance

application. The court in its final judgment, with the provisions of this Act. As appellants

besides determining the corresponding prospecting was done in violation of the law, it

compensation of the damages which may be was an illegal act and the subsequent location

caused by the prospecting, shall make a of the mining claims was also illegal and null

pronouncement as to the value and the and void. For the Mining Act regards a valid

reasonable rental for the occupation and discovery as that which gives the prospector

utilization thereof for mining purposes in case the right to locate a mining claim (Sections 29

the prospector decides to locate and exploit and 30), and the validity of a location depends

the minerals found therein." upon compliance with the law.

The purpose of the law is obvious, which is, to It is clear, of course that the validity of a
prevent trespass on private property. The location depends upon compliance with the
importance of the written permission of the statutes. The law requires that the locator
owner of private land is also apparent from shall act in good faith, and it will not
the forms prescribed by the Bureau of Mines countenance a trespass as the basis of a
for the declaration of location of a mining mining right
claim which require the locator to state that
the landowner has granted written permission SMPIs suggestion that the remedy provided in

for the prospecting and location of the mining Section 67 of the Mining Act be applied to it is

claim if the latter is located on private also unacceptable. It is evident that the

property. foregoing speaks of lease of a mining claim to


which SMPI would neither be entitled for
Section 27 is inapplicable as it never entered failure to comply with the provisions of the
the land for the purpose of "prospecting" but Mining Act and to accompany its application
already for "locating" a mining claim inasmuch for lease with a written authority of the
as the limestone deposits were prominently Landowners. In fact, SMPI left the space
exposed and spread visibly and recognizably provided for the same in its application blank.
on the surface of the land such that there was For the same reasons, authority cannot be
no need of "entering" the land. In finding the granted by the Court, nor can rental be fixed,

Midterm Cases Exclusive Jurisdiction 65


compliance with the terms of this Act being an which was passed pursuant to PD No.
indispensable prerequisite. 231 and also Municipal Ordinance No.
10 pursuant to PD No. 426, amending
A condition sine qua non is that the PD No. 231.
prospecting, exploration, discovery and
Floro Cement Corporation set up the
location must be done in accordance with the
defense that it is not liable to pay
law. As it is, SMPIs rights to use and exploit
manufacturer's and exporter's taxes
the mineral resources discovered and located
alleging among others that the
never matured because of its omission to
municipalitys power to levy and collect
comply with a condition precedent. To allow
taxes, fees, rentals, royalties or charges
SMPI its claim for surface rights and right of
of any kind whatsoever has been
way would be to countenance illegal trespass
limited or withdrawn by Section 52 of
into private property.
PD No. 463.
o Sec. 52. Power to Levy Taxes on
WHEREFORE, with the sole modification as to
Mines, Mining Corporation and
the award of temperate damages, which are
Mineral Products.Any law to the
hereby reduced as indicated, the judgment
contrary notwithstanding, no
under review is hereby affirmed in all other
province, city, municipality, barrio
respects
or municipal district shall levy
and collect taxes, fees, rentals,
royalties or charges of any kind
Floro Cement vs Gorospe whatsoever on mines, mining
claims, mineral products, or on
FACTS:
any operation, process or activity
The municipality of Lugait, province of connected therewith.
Misamis Oriental, filed a verified
CFI: Ordered Floro Cement Corporation
complaint for collection of
to pay the manufacturers and
manufacturers and exporters taxes
exporters taxes.
against Floro Cement Corporation,
engaged in the manufacture and
ISSUE & HELD: WON Ordinances Nos. 5 and
selling, including exporting, of cement.
10 of Lugait, Misamis Oriental apply to
The municipality alleged that the
petitioner Floro Corporation notwithstanding
imposition and collection of these taxes
the limitation on the taxing power of local
is based on its Municipal Ordinance No.
government as provided for in Sec. 5 of P.D.
5 (Municipal Revenue Code of 1974)
231 and Sec. 52 of P.D. 463 (Yes)

Midterm Cases Exclusive Jurisdiction 66


By the parties own stipulation of facts
RATIO: submitted before the CFI, it is admitted
Cement is not a mineral product but that Floro Cement Corporation is
rather a manufactured product. engaged in the manufacturing and
As the power of taxation is a high selling, including exporting of cement.
prerogative of sovereignty, the As such, and since the taxes sought to
relinquishment is never presumed and be collected were levied on these
any reduction or diminution thereof activities pursuant to Sec. 19 of P.D.
with respect to its mode or its rate, No. 231, Ordinances Nos. 5 and 10,
must be strictly construed, and the which were enacted pursuant to P.D.
same must be coached in clear and No. 231 and P.D. No. 426, respectively,
unmistakable terms in order that it properly apply to Floro Cement
may be applied. More specifically Corporation.
stated, the general rule is that any
claim for exemption from the tax
statute should be strictly construed People vs Abad
against the taxpayer. He who claims an
exemption must be able to point out Facts:

some provision of law creating the right;


Prior to 27 March 1978, the Director of Mines
it cannot be allowed to exist upon a
issued a commercial lease permit to one Felix
mere vague implication or inference. It
de Castro granting him the exclusive right to
must be shown indubitably to exist, for
quarry, extract and carry away sand and
every presumption is against it, and a
gravel from the Sumigar Quarry located at
well-founded doubt is fatal to the claim.
Banawe, Ifugao.
Floro Cement Corporation failed to
meet this requirement. On complaint by Felix de Castro, an
The exemption mentioned in Sec. 52 of Information was filed in the Court of First
P.D. No. 463 refers only to machineries, Instance of Ifugao, presided over by
equipment, tools for production, etc., as respondent Judge, charging private
provided in Sec. 53 of the same decree. respondents with the crime of "Theft of
The manufacture and the export of Minerals" defined and penalized under Section
cement does not fall under the said 78 of Presidential Decree No. 483, as amended
provision for it is not a mineral product. by Presidential Decree No. 1385.
It is not cement that is mined only the
mineral products composing the Respondents-accused filed a Motion to Quash
finished product. on the ground that the facts charged do not

Midterm Cases Exclusive Jurisdiction 67


constitute an offense inasmuch as they had Whether or not the facts charged in the
paid "sand and gravel tax," to the Municipal Information constitute an offense?
Treasurer of Banawe, Ifugao, for the quarrying
of sand and gravel. The taking, therefore, Ruling:

according to private respondents, was with the


Section 78. Theft of Minerals.
consent of the government. They also invoked
Any person who, without a
LOI No. 243, which allows persons to extract
mining lease or a temporary
sand and gravel even within the leased area
permit or, any other permit
for use in government infrastructures.
granted by the Secretary or the

Petitioner opposed the quashal arguing that it Director under existing mining

is error to imply that consent was given by the decrees, laws and regulations to

Government through the Municipal Treasurer mine, shall extract, remove

inasmuch as the taxes paid to the Municipal and/or dispose of minerals

Government are not the fees required by the belonging to the Government or

Bureau of Mines. from a mining claim or claims


leased, held or owned by other
On 28 January 1980, respondent Judge persons, shall be deemed to have
issued the assailed Order quashing the stolen the ores or the products
Information on the ground that violation of thereof from the mines or
P.D. No. 463 is limited to an administrative mills.
violation and that the crime of Theft under the
Revised Penal Code (Article 308) has not been The elements of the offense, therefore, are that

committed since malice, which is an essential :

element in the commission of a crime, is


(1) the accused extracted, removed and/or
lacking.
disposed of minerals;

The reconsideration prayed for by petitioner


(2) these minerals belong to the Government
was denied by respondent on 18 July 1980.
or have been taken from a mining claim or
Hence, this certiorari Petition alleging grave
claims leased, held or owned by other persons;
abuse of discretion on the part of respondent
and
Judge.

(3) the accused did not possess a mining lease


or a temporary permit or any other permit to

Issue: mine granted by the Secretary or the Director

Midterm Cases Exclusive Jurisdiction 68


under existing mining decrees, laws and receipts are insufficient evidence to prove that
regulations. the proper Government office had, in effect,
granted the required permit to extract
Evidently, the Information filed in the Court minerals from said quarry.
below includes all the foregoing elements.
Thus, it alleged (1) that the accused, The rationalization by respondent Judge that
conspiring and mutually helping one another, the taking away of sand and gravel was
wilfully and feloniously extracted, removed without malice because it was done with the
and/or disposed of minerals or material knowledge and participation of the
aggregates like sand and gravel; (2) the Government since private respondents had
minerals were taken from the Sumigar paid taxes on the sand and gravel extracted is
Quarry, Banawe, Ifugao, which is covered by a not well-taken. In crimes punished by special
commercial permit issued by the Bureau of laws, the act alone, irrespective of its motives,
Mines, Baguio City, in favor of complaining constitutes the offense.
witness Felix de Castro; and (3) the extracting
was done without any mining lease or permit WHEREFORE, the Petition is granted.

of their own pursuant to law.

It will have to be held, therefore, that based


upon the facts alleged in the Information, the
essential requisites of the Offense of "Theft of
Minerals," as specified by substantive law, are
present. Thus, respondent Judge, in
considering as evidence the three receipts of
tax payments issued by the Municipal
Cruz vs Secretary of DENR
Treasurer of Banawe, Ifugao, exceeded his
FACTS:
jurisdiction amounting to grave abuse of
Petitioners Isagani Cruz and Cesar Europa
discretion when he considered matters of
filed a suit for prohibition and mandamus as
defense extrinsic to the allegations in the
citizens and taxpayers, assailing the
Information and which should be
constitutionality of certain provisions of
substantiated during the trial.
Republic Act No. 8371, otherwise known as

Moreover, said receipts merely show payment the Indigenous Peoples Rights Act of 1997

of taxes pursuant to Provincial Ordinance No. (IPRA) and its implementing rules and

14 and not the authority to extract, remove, regulations (IRR). The petitioners assail

and/or dispose of minerals from the Sumigar certain provisions of the IPRA and its IRR on

Quarry as required by P.D. No. 463. Those the ground that these amount to an unlawful

Midterm Cases Exclusive Jurisdiction 69


deprivation of the States ownership over lands does not include the right to alienate the
of the public domain as well as minerals and same.
other natural resources therein, in violation of
the regalian doctrine embodied in section 2,
Article XII of the Constitution.

ISSUE:
Do the provisions of IPRA contravene the
Constitution?

HELD:
No, the provisions of IPRA do not contravene
the Constitution. Examining the IPRA, there is
nothing in the law that grants to the ICCs/IPs
ownership over the natural resources within
their ancestral domain. Ownership over the
natural resources in the ancestral domains
remains with the State and the rights granted
by the IPRA to the ICCs/IPs over the natural
resources in their ancestral domains merely
gives them, as owners and occupants of the
land on which the resources are found, the
right to the small scale utilization of these
resources, and at the same time, a priority in
their large scale development and exploitation.

Additionally, ancestral lands and ancestral


domains are not part of the lands of the public
domain. They are private lands and belong to
the ICCs/IPs by native title, which is a
concept of private land title that existed
irrespective of any royal grant from the State.
However, the right of ownership and
possession by the ICCs/IPs of their ancestral
domains is a limited form of ownership and

Midterm Cases Exclusive Jurisdiction 70

Anda mungkin juga menyukai